Breast Reconstruction Flashcards

1
Q

A 56-year-old woman is evaluated 6 hours after undergoing bilateral breast reconstruction with a deep inferior epigastric perforator (DIEP) flap. Doppler examination shows strong arterial signals in both flaps. The right breast appears bluish with a capillary refill time of 1 second compared to 3 seconds on the left side. Which of the following is the most appropriate next step?

A) Administration of tissue plasminogen activator
B) Application of leeches
C) Application of nitroglycerin ointment
D) Exploration in the operating room
E) Observation

A

The correct response is Option D.

The patient described has evidence of venous congestion. The reported incidence of venous congestion in free tissue breast reconstruction ranges from 2 to 20%. Causes include venous thrombosis, inadequate perforator selection, and superficial venous system dominance with lack of sufficient communication to the deep system. Signs of venous compromise include the following: cyanotic/blue color, brisker than normal capillary refill, increased tissue turgor, cooler temperature compared to normal skin (greater than 2 degrees), rapid bleeding of dark blood with pinprick, and absence of continuous venous Doppler signal. The most appropriate course of action in this scenario is emergent exploration in the operating room to assess the vascular pedicle for thrombosis, compression from hematoma, kinking, or superficial system dominance. Flap salvage rate is directly tied to timing of exploration, with higher salvage rates in flaps explored within 6 hours of identification of compromise.

Early recognition and rapid exploration of compromised flaps are the most important factors predicting flap salvage, so observation would be unacceptable. Tissue plasminogen activator is useful if diffuse clotting is suspected within the flap, but should only be given locally within the flap. Leeches can be a useful adjunct postoperatively after employing the other maneuvers described above, but would not resolve the underlying problem in this case. Application of topical nitroglycerin can improve venous congestion in random skin flaps, but has no role in the management of acute microvascular thrombosis.

How well did you know this?
1
Not at all
2
3
4
5
Perfectly
2
Q

A 45-year-old woman presents with right breast cancer and is planning a nipple-sparing mastectomy and tissue expander placement. She is specifically interested in a carbon dioxide–based expander. Which of the following is a disadvantage of this device compared with a saline tissue expander?

A) Extrusion
B) Inability to deflate
C) Increase in wound dehiscence
D) Increase in wound infection
E) Possible device dislocation

A

The correct response is Option B.

The carbon dioxide-based tissue expander (AeroForm) is a fixed-volume device and has an inability to deflate the expander.

In a prospective, multicenter, randomized controlled trial comparing carbon dioxide–based expanders and saline tissue expanders, there were no statistically significant differences in rates of wound infection, extrusion, device dislocation, or wound dehiscence. Advantages of the carbon dioxide–based expander include a more rapid expansion process and a shorter time to implant exchange. The device is self-contained and patient-controlled, so there are no needles required and possibly fewer physician office visits.

How well did you know this?
1
Not at all
2
3
4
5
Perfectly
3
Q

A 35-year-old woman presents with unilateral swelling that has developed over the past 3 months. She underwent bilateral nipple-sparing mastectomy with immediate implant reconstruction with textured, round silicone gel implants 8 years ago. Ultrasound confirms periprosthetic seroma without any masses. Which of the following is the most appropriate next step in the management of this patient?

A) Core needle biopsy
B) Fine-needle aspiration
C) Implant removal and capsulectomy
D) MRI
E) Positron emission tomography (PET) scan

A

The correct response is Option B.

The clinical scenario is concerning for breast implant–associated anaplastic large-cell lymphoma (BIA-ALCL). Aspiration of the fluid seen on ultrasonound and pathologic evaluation is necessary to confirm the diagnosis. Following the National Comprehensive Cancer Network guidelines, initial workup of an enlarged breast should include ultrasound evaluation specifically for a fluid collection, a breast mass, or enlarged regional lymph nodes (axillary, supraclavicular, and internal mammary).

MRI is appropriate for cases where ultrasound is indeterminate or requires further confirmation. This patient does not have an identifiable mass amenable to core biopsy. Positron emission tomography (PET) scan is beneficial in confirmed cases to identify associated masses, chest wall involvement, regional lymphadenopathy, and/or metastasis. Implant removal and capsulectomy is appropriate once the diagnosis is confirmed.

How well did you know this?
1
Not at all
2
3
4
5
Perfectly
4
Q

In women undergoing prosthetic breast reconstruction complicated by an expander/implant infection, which of the following is the most common gram-negative bacteria isolated from cultures?

A) Escherichia coli
B) Klebsiella
C) Proteus
D) Pseudomonas
E) Serratia

A

The correct response is Option D.

Tissue expander/implant-based breast reconstruction remains the most common form of reconstruction after mastectomy. One of the most potentially devastating complications of this form of breast reconstruction is an implant infection with need for removal of the expander/implant. The mean reported incidence of implant infection after breast reconstruction is 8%, with a range of 1 to 35%. When cultures are obtained, the most common causative bacteria on microbiology examination are gram-positive organisms (41 to 83%), specifically, Staphylococcus species (56 to 76.5%). Gram-negative bacteria accounted for 15.3 to 28.6%, with Pseudomonas (10.7 to 14%) being the most common gram-negative bacteria present on microbiology examination.

How well did you know this?
1
Not at all
2
3
4
5
Perfectly
5
Q

A 50-year-old woman presents to the clinic to discuss breast reconstruction after bilateral mastectomy. She is interested in free tissue transfer. She has a diagnosis of systemic lupus erythematosus treated with chronic steroid therapy and wants to know if she is an appropriate candidate for free flap reconstruction. Which of the following statements best describes the surgical risks for this patient with lupus compared with the general population?

A) Higher rate of free flap failure
B) Higher risk of a thromboembolic event
C) Similar rate of hernias after abdominally based free flaps
D) Similar rates of infection

A

The correct response is Option B.

The statement which best describes the surgical risks for a patient with lupus undergoing free tissue transfer for breast reconstruction is that the patient has a higher risk of a thromboembolic event than the average patient.

The rate of free flap failure in patients with lupus is similar to patients without lupus. Chronic steroid use increases the risk of wound healing complications in patients with lupus, rather than increases the risk of free flap failure. Additionally, patients with lupus have an increased risk of abdominal wall bulge and hernia after abdominally based free flaps compared with the average population. Chronic steroid use also suppresses the immune system, predisposing patients treated with steroids to increased rates of infection compared to patients not taking steroids.

How well did you know this?
1
Not at all
2
3
4
5
Perfectly
6
Q

A 53-year-old woman is evaluated for left-sided nipple reconstruction after mastectomy. She has scars on the left breast from a previous breast biopsy, as well as from the mastectomy itself. Nipple reconstruction must be designed around the scars. In single-pedicle nipple reconstruction, which of the following provides the blood supply to the pedicle?

A) Internal mammary artery perforators
B) Posterior intercostal arteries
C) Subdermal plexus
D) Superior intercostal artery
E) Thoracoacromial artery perforators

A

The correct response is Option C.

Single-pedicle nipple reconstructions, which include such techniques as the skate flap, star flap, C-V flap, and opposing tab flaps as well as other variations, create nipples from remaining mastectomy skin through adjacent tissue transfer. The flap derives its blood supply from the subdermal plexus.

The creation of the flap must keep this blood supply in mind. The flap design must avoid previous scars at the flap base and must integrate the subcutaneous fat at the base of the pedicle.

The internal mammary artery supplies the breast itself and the nipple-areola complex, and the thoracoacromial artery supplies the pectoralis muscle and the breast. The posterior intercostal arteries supply the intercostal spaces. The superior intercostal artery arises from the costocervical trunk, off of the subclavian artery, and supplies the intercostal spaces.

How well did you know this?
1
Not at all
2
3
4
5
Perfectly
7
Q

A 44-year-old previously healthy woman comes to the clinic because of a 2-week history of a painless mass in the left breast. She initially felt this mass while taking a shower. Her mother was diagnosed with fibrocystic changes. The patient denies alcohol consumption and smoking cigarettes. Examination of the left breast shows a 5-cm mobile, painless mass in the left upper external quadrant without nipple discharge, skin retractions, or color changes. Examination of a specimen obtained on biopsy discloses a phyllodes tumor, and surgical excision of the lesion is planned. Which of the following is the most important factor to prevent local recurrence after surgery?

A) Adjuvant radiotherapy
B) Concurrent axillary node dissection
C) Postoperative chemotherapy
D) Surgical margins less than or equal to 0.5 cm
E) Wide surgical margins

A

The correct response is Option E.

In a young woman who has no history of breast cancer, presents with a painless mass, and has a mammogram suggestive of fibroadenoma but a core needle biopsy showing stromal hypercellularity with atypical spindle cells and a high mitotic rate, a phyllodes tumor must be suspected.

Phyllodes tumors are uncommon fibroepithelial breast tumors that behave like benign fibroadenomas, although they have a high propensity to recur locally. More aggressive tumors can metastasize distantly. Surgery is the preferred treatment for this condition. In this context, surgical margins greater than or equal to 1 cm have been associated with a lower recurrence rate in borderline and malignant tumors.

Axillary lymph node involvement is rare. Wide local excision or mastectomy with appropriate margins is the preferred clinical intervention.

Based on limited data, the role of systemic chemotherapy in phyllodes tumors is limited. Patients with benign or borderline phyllodes tumors are usually cured with surgery and should not be offered chemotherapy unless they develop unresectable metastases.

Local recurrence rate is higher after excision with narrower margins than broader ones. The efficacy of postoperative adjuvant radiotherapy for a breast phyllodes tumor is not clear. In clinical practice, the utilization of adjuvant radiotherapy for a phyllodes tumor appears to be modest.

How well did you know this?
1
Not at all
2
3
4
5
Perfectly
8
Q

A 54-year-old woman is evaluated for nipple-areola complex reconstruction after mastectomy and silicone implant-based reconstruction. During discussion of the risks and benefits of a C-V flap, the patient asks about the long-term results of different techniques. Which of the following is the most likely long-term complication of a single-pedicle nipple-areola reconstruction?

A) Atrophic scarring
B) Delayed nipple necrosis
C) Hypertrophic scarring
D) Implant exposure
E) Loss of projection

A

) Loss of projection

The correct response is Option E.

Single-pedicle nipple reconstructions, which include techniques such as the skate flap, star flap, C-V flap, and opposing tab flaps as well as other variations, create nipples from remaining mastectomy skin through adjacent tissue transfer. The flaps derive their random-pattern blood supply from the subdermal plexus.

The creation of the flap must keep this blood supply in mind. The surgical technique must avoid previous scars at the base of the flap design and must integrate the subcutaneous fat at the base of the pedicle.

While hypertrophic and atrophic scarring can occur, they are not the most common long-term effects, and are more a function of patient characteristics than flap characteristics.

Implant exposure can occur with scar breakdown, but this is an early rather than a late complication.

Delayed nipple necrosis is technically not correct because the nipple is no longer present, and is not correct of the nipple reconstruction because necrosis of the flaps, if it occurs, usually occurs early.

How well did you know this?
1
Not at all
2
3
4
5
Perfectly
9
Q

A patient with a history of breast cancer undergoes nipple-sparing mastectomy of the right breast with immediate implant-based reconstruction. Ten months after surgery, the patient starts to recover sensitivity at the nipple. Which of the following nerves is most likely providing sensitivity to the nipple-areola complex in this patient?

A) Anterior branch of the fourth intercostal nerve
B) Anterior branch of the second intercostal nerve
C) Lateral branch of the fifth intercostal nerve
D) Lateral branch of the fourth intercostal nerve
E) Lateral branch of the second intercostal nerve
F) Lateral branch of the third intercostal nerve

A

The correct response is Option A.

The nipple and areola of the breast are innervated by both the anterior and lateral cutaneous branches of the third, fourth, or fifth intercostal nerves. The anterior and lateral cutaneous branches of the second and sixth intercostal nerves innervate breast skin only.

In anatomical studies conducted in female cadavers, the fourth intercostal nerve’s lateral cutaneous branch supplied the nipples in 93% dissected breasts. The third and fifth intercostal lateral branches were found to innervate the nipple alone in 3.6%. However, the fourth intercostal lateral branch penetrates the deep fascia in the midaxillary line, takes an inferomedial course to reach the midclavicular line, and continues through the glandular tissue towards the posterior surface of the nipple. Thus, when a mastectomy is performed, this lateral branch is the most likely one to be dissected.

On the other hand, the anterior cutaneous branches take a superficial course, as they run in the subcutaneous tissue close to the skin and reach the nipple from the lateral side. According to this, the anterior branch of the fourth intercostal nerve is most likely providing sensitivity to the nipple-areolar complex after nipple-sparing mastectomy.

How well did you know this?
1
Not at all
2
3
4
5
Perfectly
10
Q

A 55-year-old woman with a BRCA gene mutation elects to undergo bilateral mastectomy with reconstruction using a deep inferior epigastric perforator flap. BMI is 41 kg/m2. Physical examination shows both supra- and infraumbilical adiposity with excess skin and a mature cesarean delivery scar. This patient has the highest risk for which of the following early postoperative complications?

A) Abdominal wall bulge
B) Abdominal wall hernia
C) Delayed wound healing of donor site
D) Fat necrosis of the flap
E) Flap failure

A

The correct response is Option C.

The highest risk is for delayed wound healing of the donor site. Because of the patient’s morbid obesity and prior cesarean delivery scar, she has the highest risk for some form of wound breakdown or prolonged wound healing. This risk can be as high as 50 to 60% for morbidly obese patients. These trends are similar in patients following reconstruction with a transverse rectus abdominis musculocutaneous (TRAM) flap.

To reduce these risks, minimal undermining is recommended and only done if necessary. Techniques to preserve all cutaneous perforators will help reduce the risk associated with closure of the donor sites.

While morbid obesity can be associated with increased abdominal wall thickness, there is no correlation with the occurrence of abdominal wall bulge or hernia. The risks for these complications are less than 2%.

Patients with morbid obesity can have shorter operative times, but there is no correlation with overall flap failure, with rates reported to be less than 1%. This is also seen in pedicled and free TRAM flap reconstructions.

Rates of fat necrosis of the flap can be as high as 10 to 15% in patients undergoing reconstruction, but this risk is not affected by body habitus or body mass index and is lower than the risk for delayed wound healing.

How well did you know this?
1
Not at all
2
3
4
5
Perfectly
11
Q

A healthy 45-year-old woman with a history of breast malignancy underwent bilateral mastectomy and reconstruction with tissue expanders followed by exchange for cohesive silicone gel implants eight years ago with routine postoperative MRI surveillance. She comes to the office to report pain and tightness in the right breast that has gradually increased over the past month. On examination, temperature is 36.8°C (98.2°F), blood pressure is 112/76 mmHg, and heart rate is 68 bpm. The right breast appears fuller than the left breast; otherwise, the right implant is in a symmetric position with the left side. The skin is otherwise normal in appearance, and there is no tenderness on palpation. Which of the following is the most appropriate next step in management?

A) MRI of the right breast to assess the integrity of the implant
B) One week of an oral antibiotic and prednisone taper
C) Operative exploration, culture, and replacement of implant
D) Referral of the patient back to her medical and surgical oncologists
E) Ultrasound of the right breast and fine-needle aspiration of any fluid

A

The correct response is Option E.

Patients that present with a late seroma should be evaluated for possible Breast implant associated Anaplastic Large Cell Lymphoma (BI-ALCL). A late seroma is usually accepted as occurring 1 year following surgery: however there are cases of BI-ALCL seroma that have presented as early as 4 months.

The first step in evaluation for BI-ALCL is an ultrasound followed by fine needle aspiration is indicated. The fluid requires evaluation beyond routine cell cytology. Immunohistochemistry test for CD30 was the most commonly positive marker for BI-ALCL. Immunohistochemistry stains specific antigens in cells by binding to this antigen in an antibody/antigen reaction. The specific stain can then be seen under light microscopy. CD30 antibody labels anaplastic large cell lymphoma cells. CD30 is a transmembrane cytokine receptor belonging to the tumor necrosis factor receptor family and characteristically stains ALCL cells.

MRI for implant integrity and referral to her Oncologist may be needed but it is not the most appropriate next step. BIA-ALCL needs to be ruled out. Immediate operative exploration is not indicated before fluid aspiration and immunohistochemistry evaluation. Antibiotics and prednisone is not indicated in this patient without evidence of infection or inflammation (red breast syndrome).

How well did you know this?
1
Not at all
2
3
4
5
Perfectly
12
Q

An otherwise healthy 54-year-old perimenopausal woman is scheduled for a mastectomy for biopsy-proven right-sided grade 2 ductal carcinoma. According to the National Comprehensive Cancer Network (NCCN) guidelines, postmastectomy radiation therapy will be the standard of care for this patient if she has which of the following surgical outcomes?

A) 1-cm surgical margins, four positive axillary lymph nodes
B) 1-cm surgical margins, one positive axillary sentinel node
C) 1-mm surgical margins, no positive axillary nodes
D) 5-mm surgical margins, no positive axillary nodes
E) 5-mm surgical margins, three positive axillary nodes

A

The correct response is Option A.

Traditionally, the need for radiation therapy has been a contraindication for implant-based reconstruction, and autologous reconstruction is the conservative gold standard for women with advanced cancer needing postmastectomy radiation. More recently, there have been reports of successful implant based reconstruction in the setting of postmastectomy radiation that have similar complication profiles and good oncologic outcomes compared with autologous reconstruction. Protocols vary between those that radiate the expander and then expand, and those that expand and then radiate the permanent implant. Being able to anticipate which patient will require postmastectomy radiation is essential for joint decision making about breast reconstruction with the patient prior to her mastectomy.

By National Comprehensive Cancer Network (NCCN) guidelines, relative indications for postmastectomy radiation therapy include: positive sentinel node with unknown status of other axillary nodes, one to three positive nodes on permanent histology, and close surgical margins (less than 5 mm). Postmastectomy radiation is recommended as the standard of care in the situations of positive surgical margins with the inability to get clear margins and four or more positive lymph nodes.

How well did you know this?
1
Not at all
2
3
4
5
Perfectly
13
Q

A 44-year-old woman presents in evaluation for breast reconstruction with biopsy-proven left breast-infiltrating ductal carcinoma after routine mammography discovered a 7-cm lesion. She has been referred to medical oncology and genetic testing is pending. Her past medical history is significant for hypertension and scleroderma. On examination, she has grade I ptosis and wears a size 34A brassiere. During the consultation, the patient reports a strong preference for lumpectomy and oncoplastic reconstruction over total mastectomy. Which of the following is most likely to increase this patient’s chances of qualifying for breast-conserving therapy?

A) Active scleroderma
B) BRCA-1 gene mutation
C) Multicentric tumor
D) Preoperative chemotherapy
E) Small-sized breasts

A

The correct response is Option D.

Preoperative chemotherapy could increase this patient’s chances of qualifying for locoregional treatment (partial mastectomy or lumpectomy). Studies have shown that breast conservation rates are improved with preoperative systemic therapy, which can also render inoperable tumors resectable. Other potential benefits of preoperative (neoadjuvant) chemotherapy include providing important prognostic information based on response to therapy, minimizing the extent of axillary surgery, and allowing time for genetic testing and reconstructive planning prior to surgery. A small-sized breast would likely provide insufficient uninvolved breast tissue for breast-conserving therapy after resection of a large (7 cm) mass. The same applies to multicentric tumors.

Whole breast irradiation is strongly recommended after lumpectomy, with studies showing a favorable effect in reducing the 10-year risk of recurrence (19% versus 35%) and the 15-year risk of breast cancer death (21% versus 25%). Therefore, patients with (relative) contraindications to radiation therapy, such as lupus or scleroderma (connective tissue disease involving the skin), should ordinarily be offered total mastectomy, particularly if this resolves the need for radiation therapy. While radiation therapy would likely still be considered for this particular patient even after total mastectomy (tumor size greater than 5 cm), the diagnosis of scleroderma itself does not increase her chances of qualifying for breast conservation surgery. BRCA-1 gene mutation and other genetic predispositions to breast cancer are relative contraindications for breast-conserving therapy. These patients may be considered for prophylactic bilateral mastectomy for risk reduction.

How well did you know this?
1
Not at all
2
3
4
5
Perfectly
14
Q

A 41-year-old woman presents with right breast lobular carcinoma in situ (LCIS) involving a 1-cm area with no palpable axillary nodes. According to the TNM staging system, which of the following is this patient’s T classification?

A) Tx
B) Tis
C) T0
D) T1a
E) None; there is no TNM staging for LCIS

A

The correct response is Option E.

Lobular carcinoma in situ (LCIS) has been removed from the staging classification system in the 8th edition and is no longer included in the pathologic tumor in situ (pTis) category. LCIS is treated as a benign entity with an associated risk for developing carcinoma in the future but not as a malignancy capable of metastases. There is a small subset of LCIS that has high-grade nuclear features and may exhibit central necrosis. This subset has been referred to as pleomorphic LCIS and has histologic features that partially overlap the features of ductal carcinoma in situ (DCIS), including the potential to develop calcifications detectable by mammography. The expert panel debated whether to include this variant of LCIS in the pTis category; however, there are insufficient data in the literature regarding outcomes and reproducible diagnostic criteria for this LCIS variant. Cases exhibiting DCIS and LCIS are classified as pTis (DCIS).

How well did you know this?
1
Not at all
2
3
4
5
Perfectly
15
Q

A 19-year-old woman with a medical history significant for Poland syndrome and a BMI of 19 kg/m2 undergoes first-stage breast reconstruction with a tissue expander that is complicated by extrusion and infection 40 days after implantation. A photograph is shown. Attempts at implant salvage are made. The presence of which of the following factors is most likely to lead to decreased salvage rates?

A) BMI of 19 kg/m2
B) Culture-positive Staphylococcus sp
C) Hemoglobin A1c of 6.5%
D) Prepectoral placement of the device
E) Use of acellular dermal matrix

A

The correct response is Option B.

It has been shown that successful breast device salvage in breast reconstruction is possible if caught early. However, there are associated factors with failure, including culture-positive Staphylococcus (epidermidis or aureus), as demonstrated by several studies. Other associated risk factors for failure include obesity, poorly-controlled diabetes, smoking, history of radiation therapy, postoperative seroma, and early contamination of the implant with biofilm formation. Therefore, prompt and aggressive intervention is warranted in these situations where the device is threatened by either infection and/or exposure. This includes both surgical and antimicrobial options.

In a 2017 study, prepectoral and subpectoral placement demonstrated comparable complications. Acellular dermal matrix did not increase failure rates.

How well did you know this?
1
Not at all
2
3
4
5
Perfectly
16
Q

A 42-year-old woman with a history of a cesarean delivery from a low-transverse abdominal incision is scheduled to undergo a unilateral deep inferior epigastric perforator (DIEP) flap breast reconstruction. BMI is 28 kg/m² and the distance from nipple to sternal notch is 24 cm per side. This patient’s history of cesarean delivery is most likely to have which of the following effects?

A) Decreased abdominal seroma
B) Decreased flap venous congestion
C) Increased flap arterial thrombosis
D) Increased flap fat necrosis
E) No overall effect

A

The correct response is Option B.

Pfannenstiel incisions are the preferred access for cesarean deliveries. They are not a contraindication for abdominal-based flaps for breast reconstruction because the deep inferior epigastric circulation is not disturbed. However, the superficial epigastric circulation may be divided. The net result appears to be a more robust venous circulation with a protective effect against fat necrosis in the flap. This venous division causes a delay-type phenomenon—during healing increased branches are formed from the superficial epigastric circulation, and there is some evidence of new connections to the venae comitantes of the deep epigastric venous circulation.

There is evidence, however, for an increased rate of abdominal healing problems, including seroma (15% versus 6%), wound healing problems, and fat necrosis in the abdomen. There is no evidence for an effect on the arterial circulation of the flap.

How well did you know this?
1
Not at all
2
3
4
5
Perfectly
17
Q

A 50-year-old woman comes to the office 6 weeks after undergoing right mastectomy and immediate placement of a tissue expander. She reports swelling and redness of the right breast. A photograph is shown. Which of the following factors is most predictive of implant salvage failure in this patient?

A) Culture positive for Pseudomonas species
B) Elevated body mass index
C) Periprosthetic seroma
D) Presence of cellulitis
E) Previous irradiation

A

The correct response is Option A.

Immediate implant-based reconstruction has become increasingly popular over the past two decades, accounting for over 70% of all reconstructions in the United States. The benefits of immediate reconstruction are numerous, including decreased recovery/number of required procedures and increased patient psychological well-being and aesthetic outcome. However, the complication (seroma, mastectomy flap necrosis, loss of implant, and infection) rates after implant-based reconstruction remain relatively high. Infection rates in the reported literature range from 2.5 to 24%.

Historically, periprosthetic infection or implant exposure mandated immediate implant removal. However, numerous studies over the past several decades have demonstrated implant salvage rates of 37.3 to 73% depending on the methods employed. Several studies have looked at the predictive factors that increase the risk of a failed salvage attempt. Salvage was typically defined as administration of systemic antibiotics (oral or intravenous), removal of the infected implant, partial/total capsulectomy, pocket curettage, implant pocket irrigation with antibiotic solution, and placement of a new device.

Factors associated with implant salvage failure include an elevated white blood cell count, elevated temperature, deep-seated pocket infection (purulent periprosthetic fluid), and atypical pathogens such as methicillin-resistant Staphylococcus aureus (MRSA) and Pseudomonas species. Spear et al. showed that 93.9% of mild implant infections (localized cellulitis) could be salvaged compared with a 30% salvage rate in the severe infection group. Factors such as smoking, chemotherapy, previous irradiation, mastectomy skin necrosis, increased BMI, and use of acellular dermal matrix (ADM) have demonstrated increased rates of implant-related infections, but these factors have not been demonstrated to increase the risk of implant salvage failure.

How well did you know this?
1
Not at all
2
3
4
5
Perfectly
18
Q

A 68-year-old woman comes to the office for a delayed breast reconstruction. She had right breast cancer and a mastectomy followed by chemotherapy and radiation therapy 1 year ago. BMI is 35 kg/m2. Medical history includes well-controlled type 2 diabetes mellitus, a previous cesarean section through a low transverse incision, and an open cholecystectomy through a subcostal incision. The patient requests autologous reconstruction, but the surgeon is not comfortable performing a free flap. Which of the following is the most appropriate method for reconstruction?

A) Bipedicled transverse rectus abdominis musculocutaneous (TRAM) flap
B) Contralateral pedicled TRAM flap
C) Ipsilateral pedicled TRAM flap
D) Surgical delay procedure followed by contralateral pedicled TRAM flap
E) Surgical delay procedure followed by ipsilateral pedicled TRAM flap

A

The correct response is Option D.

In this obese patient with right breast cancer and a previous subcostal incision, a delay procedure with a contralateral transverse rectus abdominis musculocutaneous (TRAM) flap is the most appropriate method for reconstruction. Although a contralateral TRAM flap can be performed without a delay procedure, it has been shown that the addition of a delay procedure decreases risks of ischemia to the flap. In addition, a delay procedure may also reduce risks of abdominal wall complications. In this patient, the subcostal incision excludes right-sided pedicled flap reconstruction, so an ipsilateral pedicled TRAM would not be the right choice, nor would a bipedicled TRAM flap.

Although there is controversy on which patients should have a delay procedure, the use of this technique has usually been limited to high-risk patients and to those requiring large amounts of tissue. Multiple reports have shown that obese patients undergoing a delay can decrease the risks of tissue related ischemia. In a paper by Wang et al., the delay procedure was performed at least 14 days prior to the reconstruction; however, other studies have shown improvements at 7 days. The procedure described consists of ligation of both deep inferior epigastric arteries and veins bilaterally accessed from an inferior flap incision. This can also be done laparoscopically. Some authors advocate more extensive incisions and elevating portions of the flap; however, there is little data to show that this is effective or necessary.

How well did you know this?
1
Not at all
2
3
4
5
Perfectly
19
Q

A 52-year-old woman undergoes autologous breast reconstruction with unilateral deep inferior epigastric perforator (DIEP) flaps. According to the Hartrampf model of perfusion zones, if the lateral row perforator vessels are used, in which chronological order will the flap zones be perfused?

A) I – II – III – IV
B) I – III – II – IV
C) II – I – III – IV
D) II – I – IV – III
E) IV – III – II – I

A

The correct response is Option B.

In medial perforator-based flaps, the zones are perfused in the order I – II – III – IV (A) as shown in the image. In lateral perforator-based flaps, however, the zones are perfused in the order I – III – II – IV (B).

How well did you know this?
1
Not at all
2
3
4
5
Perfectly
20
Q

A 64-year-old woman who is postmenopausal asks why she has not been prescribed hormone replacement therapy with estrogen and progestin like her mother was. Supplementation with these hormones is associated with an increased risk for which of the following?

A) Coronary artery disease
B) Diabetes
C) Endometrial cancer
D) Invasive breast cancer
E) Osteoporosis

A

The correct response is Option D.

Hormone replacement therapy has fallen out of favor because of a risk profile that is believed to exceed the potential benefits. Combined estrogen and progestin supplementation is thought to be associated with an increased risk for invasive breast cancer but may decrease the risk for diabetes and osteoporosis. It is thought to not impact the risk for coronary artery disease or endometrial cancer.

How well did you know this?
1
Not at all
2
3
4
5
Perfectly
21
Q

A 46-year-old woman who is 5 ft 7 in (170 cm) tall and weighs 135 lbs (61 kg) is evaluated one year following bilateral nipple-sparing mastectomy and immediate reconstruction with placement of 350-mL smooth, round silicone gel implants beneath the pectoralis major muscle. Since the surgery, she has experienced hyperdynamic deformity of her breasts. On physical examination, the breast reconstruction appears natural, and there is significant movement of the breasts when the patient flexes her chest. Which of the following is the most appropriate management for this patient?

A) Inject botulinum toxin into the pectoralis major muscle
B) Inject triamcinolone-40 into the areas of tenderness using ultrasound guidance
C) Move the implants to the prepectoral plane and cover them fully with acellular dermal matrix
D) Perform a breast MRI to assess for rupture of the implants
E) Refer the patient to a physical therapist for range of motion, massage, and ultrasound treatments

A

The correct response is Option C.

This patient is experiencing significant movement because her implants were placed beneath the pectoralis major muscles. While reconstruction options are limited in this otherwise healthy and very thin patient who is not a good candidate for fat grafting or pedicled or free tissue transfer, placing implants over the pectoralis major muscles and covering the implants fully with acellular dermal matrix would be the most appropriate method of reconstructing her breasts and addressing her concerns.

Physical therapy and muscle relaxants are unlikely to produce long-term improvement. An MRI would likely be nondiagnostic, and even if her implants were ruptured, change to a prepectoral plane is still indicated. Botulinum toxin type A is likely not as effective for long-term significant improvement as reoperation. Triamcinolone would not be effective for hyperdynamic deformity.

How well did you know this?
1
Not at all
2
3
4
5
Perfectly
22
Q

The capsules from patients with breast implant–associated anaplastic large-cell lymphoma (ALCL) have significant presence of which of the following bacteria?

A) Escherichia coli
B) Ralstonia pickettii
C) Staphylococcus aureus
D) Pseudomonas aeruginosa
E) Serratia marcescens

A

The correct response is Option B.

Most concerning in the past two decades is the incidence of breast implant–associated anaplastic large-cell lymphoma (ALCL). This entity was first diagnosed and associated with breast implants in 1997, and is almost only associated with a history of textured implants and/or tissue expanders. The most common presentation of these patients is late seroma, with some patients presenting with mass, tumor erosion, or lymph node metastasis. A recent review of the world literature on this entity include the following: (1) 173 cases were documented, (2) no cases were found in patients with documented smooth devices only (although this remains controversial, as the data in many cases are incomplete), (3) there may be an associated genetic predisposition as suggested for cutaneous T-cell lymphoma, and (4) the cause is likely multifactorial.

Bacterial biofilm is thought to be an inciting factor for the development of both breast-implant related ALCL and Non-Tumor related capsule contractures. The capsules from patients with tumor had significant presence of Gram-negative bacteria (Ralstonia species) compared to nontumor capsules (Staphylococcus species). Such data may support the bacterial induction model, as there are also other types of implant-associated lymphomas.

How well did you know this?
1
Not at all
2
3
4
5
Perfectly
23
Q

A 45-year-old woman who is obese is considering unilateral mastectomy and reconstruction of the left breast because of invasive ductal carcinoma. Which of the following patient characteristics is associated with the lowest risk for complications from a nipple-sparing mastectomy?

A) BMI of 41 kg/m2
B) Grade III ptosis of the breast
C) Nipple retraction
D) Tumor distance from nipple of 5 cm
E) Tumor size of 6 cm

A

The correct response is Option D.

Nipple-sparing mastectomy is increasing in popularity. To decrease the risk for surgical complications as well as oncologic complications, smaller tumors located further from the nipple in patients without morbid obesity or severe ptosis are considered better candidates for treatment with nipple-sparing mastectomy. Clinical involvement of the nipple, including retraction, would suggest that nipple-sparing mastectomy should not be performed.

How well did you know this?
1
Not at all
2
3
4
5
Perfectly
24
Q

Which of the following is the most likely chronic effect of post-mastectomy radiation therapy?

A) Desquamation
B) Dyspigmentation
C) Edema
D) Erythema
E) Ulceration

A

The correct response is Option B.

Radiation therapy induces tissue injury that can be categorized as acute or chronic. The spectrum of acute injury includes erythema, edema, desquamation, hyperpigmentation, and ulceration, ranging from mild to severe. Acute radiation dermatitis occurs in upward of 85% of treated patients. Chronic injury involves skin atrophy, dryness, telangiectasia, dyspigmentation, and dyschromia. In the breast, it leads to chronic fibrosis of the skin and subcutaneous tissues. This fibrosis and surrounding injury can lead to pain and restricted movement of the arm. The chronic changes from radiation can take months to years to fully manifest.

How well did you know this?
1
Not at all
2
3
4
5
Perfectly
25
Q

A 43-year-old woman who is BRCA-positive is scheduled to undergo bilateral mastectomy. Tissue expander–based reconstruction is planned. Which of the following is the optimal duration of antibiotic prophylaxis for this patient?

A) No preoperative antibiotic
B) One preoperative antibiotic dose and another dose during skin closure
C) One preoperative antibiotic dose, followed by 24 hours of treatment while in the hospital
D) One preoperative antibiotic dose, followed by 24 hours of treatment while in the hospital and then discharge on oral antibiotics until drains are removed
E) One preoperative antibiotic dose, followed by 24 hours of treatment while in the hospital and then maintenance on oral antibiotics until tissue expanders are exchanged

A

The correct response is Option C.

The overall complication rate in breast reconstructive surgery is as high as 60%. Infection rates can exceed 20%, much higher than in clean elective surgery. The CDC guidelines suggest only 24 hours of peri-operative antibiotics beginning thirty minutes prior to skin incision. However, not all plastic surgeons agree with this. A 2013 meta-analysis found when comparing combined patient cohorts receiving no antibiotics, antibiotics for less than 24 hours, and antibiotics for greater than 24 hours, the average infection rates were 14.4, 5.8, and 5.8%, respectively. This demonstrated that the administration of antibiotics made a difference, however duration beyond 24 hours did not.

A study was published in 2013 evaluating the difference in surgical site infection between two different prophylactic antibiotic durations (24 hours and until drain removal). In this prospective, randomized, controlled non-inferiority trial, 24 hours of antibiotics is equivalent to extended oral antibiotics for surgical-site infection in tissue expander immediate breast reconstruction patients.

How well did you know this?
1
Not at all
2
3
4
5
Perfectly
26
Q

A 45-year-old woman comes to the office for consultation about immediate bilateral breast reconstruction of a right-sided tumor measuring 2.5 cm. Biopsy reveals a HER-2/neu negative invasive ductal carcinoma without lymphovascular invasion. The patient requests nipple-sparing mastectomy. Physical examination shows a palpable mass is located in the right upper outer quadrant approximately 1.5 cm from the nipple-areola complex and is freely mobile. There is no lymphadenopathy on exam. Based on current literature, which of the following best describes this patient’s candidacy for the requested procedure?

A) Good candidate based on current presentation
B) Not a candidate because of lymph node status
C) Not a candidate because of tumor location
D) Not a candidate because of tumor pathology
E) Not a candidate because of tumor size

A

The correct response is Option C.

Nipple-sparing mastectomy or total skin-sparing mastectomy is becoming an increasingly popular choice for women because of the excellent cosmetic outcomes and the ability to save the nipple-areola complex that may provide psychological benefits with increased patient satisfaction as well. Nipple-sparing mastectomy appears to be oncologically safe with low risks of cancer recurrence in the literature thus far. However, there has been little long-term follow-up, so this approach is still somewhat controversial because the oncologic safety and locoregional recurrence have not been examined definitively. Although certain centers are pushing the envelope regarding the use of this technique in a wide range of patients, the current literature supports the following exclusion criteria:

A. Tumor size greater than 5 cm B. Tumor location less than 2 cm from the nipple C. Evidence of axillary disease D. Tumor involvement on retroareolar biopsy E. Lymphovascular invasion, human epidermal growth factor receptor-2 positivity, and/or HER-2/neu positivity on biopsy

The current patient’s tumor was found to have a tumor-to-nipple distance of 1.5 cm which is a relative contraindication to nipple-sparing mastectomy in this case.

How well did you know this?
1
Not at all
2
3
4
5
Perfectly
27
Q

A 54-year-old woman undergoes bilateral immediate tissue expander–based breast reconstruction. BMI is 36 kg/m2. On postoperative day 10, examination shows bilateral breast erythema; empiric antibiotics are started. In addition to Staphylococcus species, which of the following bacteria should be treated as the next most likely pathogen?

A) Bacteroides fragilis
B) Enterococcus faecalis
C) Mycobacterium marinum
D) Pasteurella multocida
E) Pseudomonas aeruginosa

A

The correct response is Option E.

After Staphylococcus aureus and S. epidermidis, Pseudomonas aeruginosa is among the the next most common sources of breast infections. It is common in hospitalized or immunocompromised patients, as well as patients with foreign body devices such as catheters or implants. It is a gram negative rod, and common antibiotic treatments include advanced β-lactams (piperacillin, ceftazidime), carbapenems, quinolones, and aminoglycosides. Dual coverage is often recommended in severe infections. In the case of breast implant infections, if there is not rapid improvement on antibiotic therapy, or if significant systemic symptoms develop (vital sign instability, high white blood cell count, fever, renal impairment), then surgical washout and device removal is mandatory. In patients without systemic symptoms, wash out and new implant placement can be an option in carefully selected and counseled patients. Other breast pathogens include Escherichia coli, Propionibacterium, and Corynebacterium.

More than 300,000 breast implant procedures are performed each year in the United States. In reconstructive cases, the infection rate averages 6% and the explantation rate 3% (range, 1.5 to 8%). Preventative measures include proper patient selection, preoperative MRSA management when carriers are suspected, routine presurgery chlorhexidine washes, proper antibiotic timing presurgery and continuation of antibiotics in implant reconstruction cases for at least 24 hours (though the optimal treatment duration has not yet been determined).

None of the other bacteria listed are common in breast infections, though all are common pathogens. Bacteroides are anaerobic gram-negative rods that are common in gut flora and feces.

Enterococcus faecalis is a frequent cause of nosocomial infection, with a high prevalence of multi-drug resistance. It is a gram-positive coccus, and is not commonly seen in breast surgery patients as it primarily colonizes the digestive tract.

Mycobaterium marinum is a rare pathogenic cause of hand infections from injuries that occur in aquatic environments.

Pasteurella multocida is a frequent cause of animal bite infections, particularly from cats and dogs.

How well did you know this?
1
Not at all
2
3
4
5
Perfectly
28
Q

A 62-year-old woman is evaluated because of a new 2 × 2-cm open area near her left axillary fold. Medical history is significant for left breast cancer previously treated with bilateral mastectomies, left axillary node dissection, and adjuvant chemoradiation 10 years ago. A photograph is shown. She has been compliant with postoperative oncologic surveillance and has had no recent trauma. Which of the following underlying conditions is most likely responsible for her current presentation?

A) Empyema with spontaneous drainage
B) Lymphedema drainage tract
C) Osteoradionecrosis of the underlying rib(s)
D) Recurrent breast cancer
E) Skin ulceration from intertriginous shearing forces

A

The correct response is Option C.

The effects of ionizing radiation are permanent and may present either acutely or in delayed fashion, even years after the original radiation insult. The mechanism of injury from this radiation is through free radical production which, in turn, directly damages the DNA. In the acute period, the effects of radiation may manifest themselves as erythema and edema of the skin, vasodilation with endothelial edema, and lymphatic obliteration. This eventually leads to capillary thrombosis and subsequent inadequate tissue oxygenation. Over time, nonhealing ulcers can spontaneously develop, sometimes years later.

Although recurrent cancer is always a concern in patients with a personal history of cancer, proper, regular, and thorough surveillance can often detect recurrences early, especially in compliant patients. Most recurrences occur within the first 5 years.

Abscesses usually present initially with pain, erythema, and localized fluctuance, and often with associated fever and/or malaise. Spontaneous necessitation to the skin would also result in purulent drainage.

Lymphedema can be a chronic condition after mastectomy and axillary node dissection, and is usually manifested as generalized edema of the ipsilateral upper extremity. Sinus tract formation is rare.

Intertriginous shearing would most often present as superficial epidermal loss with possible superinfection with yeast due to moisture.

How well did you know this?
1
Not at all
2
3
4
5
Perfectly
29
Q

A 48-year-old woman is evaluated for bilateral microsurgical breast reconstruction. Compared with the deep inferior epigastric perforator (DIEP) flap, the superficial inferior epigastric artery (SIEA) flap places the patient at a greater risk for which of the following complications?

A) Abdominal bulge
B) Donor site dehiscence
C) Fat necrosis
D) Flap failure
E) Umbilical necrosis

A

The correct response is Option D.

Several comparative studies have reported a higher incidence of anastomotic thrombosis and failure with the superficial inferior epigastric artery (SIEA) flap than with flaps based on the deep inferior epigastric artery (DIEA). These failure rates range from 7.35 to 14%. Most of these failures were arterial in nature. Since SIEA flaps do not require an incision into the anterior rectus sheath or rectus muscle, bulges do not occur. Reported fat necrosis rates are similar between SIEA and DIEA flaps. There is no evidence for a difference in donor site dehiscence or umbilical necrosis rates.

How well did you know this?
1
Not at all
2
3
4
5
Perfectly
30
Q

A 48-year-old woman undergoes immediate unilateral breast reconstruction with a free deep inferior epigastric artery perforator (DIEP) flap. At the conclusion of the procedure, the flap skin paddle is noted to have venous congestion. Upon reexploration, the venous anastomosis appears patent with venous outflow detected by handheld pencil Doppler evaluation, but the flap continues to have venous congestion with brisk capillary refill. Which of the following is the most appropriate next step in management?

A) Apply leeches postoperatively
B) Loosely re-inset the flap and monitor closely
C) Perform a second venous anastomosis using the superficial inferior epigastric vein
D) Perform a second venous anastomosis using the vena comitans
E) Revise the venous anastomosis using a hand-sewn technique

A

The correct response is Option C.

Preservation of the superficial inferior epigastric veins (SIEV) during flap harvest is a useful preventive measure in microsurgical free tissue transfer operations. These veins can serve as important lifeboats to augment venous outflow in the setting of venous congestion. Typically, if a free flap demonstrates venous congestion, the inset should be taken down and the pedicle, recipient vessels, and anastomoses should be interrogated. Simple issues, such as mechanical compression or twisting of the vein, should be ruled out. Next, the SIEV should be inspected. If engorged, the flap is likely reliant on superficial outflow, and this vein should be connected to a recipient vessel to augment the venous outflow of the flap. Options for recipient veins include an anterograde branch on the pedicle vena comitans, or in a retrograde fashion to the vena comitans that was not used in the initial set of anastomoses.

In this case scenario, the flap continued to demonstrate venous congestion intraoperatively. This makes it unlikely that tension or pressure from the inset of the flap was causing the venous outflow obstruction. Furthermore, leech therapy is not indicated for a free flap with global venous congestion.

The venous coupling device is safe and effective for the anastomosis of veins in DIEP flap surgery. It has not been associated with patency rates that are different from hand-sewn anastomoses. The coupling device, however, has been shown to reduce the microsurgery time.

The use of one or two veins in microsurgical free tissue transfer is a topic that has been debated for several years. While some studies indicate that the use of two venous connections may reduce the velocity of blood flow across the anastomosis, there is not sufficient data to support differences in flap outcomes or thrombotic events. Therefore, the routine use of a second vein is largely up to surgeon preference.

How well did you know this?
1
Not at all
2
3
4
5
Perfectly
31
Q

A 43-year-old woman undergoes the second stage of tissue expander–based breast reconstruction. Exchange of the tissue expander for a smooth round silicone implant is planned along with a superior capsulotomy and fat grafting to the upper pole for contour improvement. Which of the following is an increased risk associated with fat grafting to the breast in this patient?

A) Anaplastic large cell lymphoma
B) Benign lesions
C) Hypopigmentation
D) Infection
E) Recurrent breast cancer

A

The correct response is Option B.

Autologous fat grafting is a widely accepted technique in breast reconstruction. A large systematic review recently confirmed the oncologic safety of this technique but did report a significant incidence of benign lesions including cysts and calcifications. Fat grafting is not associated with an increased risk of recurrent breast cancer, infection or hypopigmentation.

How well did you know this?
1
Not at all
2
3
4
5
Perfectly
32
Q

A 54-year-old woman with breast cancer undergoes a skin-sparing mastectomy with tissue expander reconstruction. Adjuvant chemotherapy and subsequent radiation therapy have been recommended. Chemotherapy proceeds during tissue expansion. Radiation may be performed either before or after the implant exchange procedure. When compared with radiating the permanent implant, radiating the tissue expander is most likely to increase the risk of which of the following?

A) Cancer recurrence
B) Capsular contracture
C) Device rupture
D) Explantation
E) Radiation dermatitis

A

The correct response is Option D.

Cordeiro et al. updated the largest series of women undergoing two-stage implant breast reconstruction who require postmastectomy radiation. The authors found that radiating the tissue expander, as opposed to the permanent implant, increased the rate of reconstructive failure by 46%. However, aesthetic results were better and capsular contracture was less frequent. There were no differences in patient-reported outcomes. This study confirmed the earlier findings of Nava et al. regarding reconstruction failure (explantation). There are no known differences in cancer recurrence between the two approaches discussed.

How well did you know this?
1
Not at all
2
3
4
5
Perfectly
33
Q

A 52-year-old woman comes to the office to discuss revision of breast reconstruction following mastectomy for breast cancer. She is undergoing adjuvant treatment with an agent that interferes with her body’s natural mechanisms that promote native breast growth, but she cannot remember its name. She is most likely being treated with which of the following agents?

A) Alkylating agent (cyclophosphamide)
B) Anthracycline (doxorubicin)
C) Aromatase inhibitor (anastrozole)
D) Platinum agent (cisplatin)
E) Taxane (paclitaxel)

A

The correct response is Option C.

Aromatase inhibitors such as anastrozole impair conversion of androgens to estrogens. Estrogens promote normal breast tissue growth as well as growth of many breast cancers. The other options (taxanes, anthracyclines, alkylating agents, and platinum agents) are all chemotherapeutic agents that do not particularly target hormones involved in normal breast growth mechanisms.

How well did you know this?
1
Not at all
2
3
4
5
Perfectly
34
Q

Improvement in which of the following is an advantage of nipple-areola complex reconstruction?

A) Breast feeding ability
B) Breast mound shape
C) Reactivity to touch
D) Satisfaction with reconstruction
E) Sexual sensation

A

The correct response is Option D.

Satisfaction with reconstruction, quality of life, and feeling of completeness with reconstruction have all been shown to be linked to nipple and/or areola reconstruction. The reconstruction does not restore nipple function; therefore, breast feeding, sexual sensation, and reactivity to touch are not accomplished with nipple and/or areola reconstruction. Often the nipple reconstruction, if using local tissue techniques, may actually flatten the anterior aspect of the breast and is not thought to positively impact the breast mound shape.

How well did you know this?
1
Not at all
2
3
4
5
Perfectly
35
Q

A 35-year-old woman with a Stage T2 infiltrating ductal carcinoma is scheduled to undergo a skin-sparing, right total mastectomy and a nipple-sparing, left prophylactic mastectomy. The possibility of adjuvant radiation therapy to the right breast depends on the final surgical pathology. The patient has a history of smoking. BMI is 28 kg/m2. She wears a brassiere with a D cup and would like the postoperative result to be of a similar size. Which of the following immediate bilateral reconstructive techniques is most appropriate for this patient?

A) Abdominal-based free flaps
B) Gluteal-based free flaps
C) Latissimus dorsi myocutaneous flaps and silicone implants
D) Silicone implants and acellular dermal matrix
E) Tissue expanders and acellular dermal matrix

A

The correct response is Option E.

For this patient in whom postoperative radiation therapy is possible, the best first-stage, immediate reconstructive approach is placement of tissue expanders with acellular dermal matrix. The outcome of immediate autologous flap reconstruction may be compromised if subjected to adjuvant radiation therapy and is best delayed until after such treatment has been rendered. Although successful, cost-effective outcomes are possible with a single-stage, direct-to-implant approach, this patient has risk factors for early revision and implant failure due to her large breasts and history of smoking.

How well did you know this?
1
Not at all
2
3
4
5
Perfectly
36
Q

After a nipple-sparing mastectomy, which of the following branches of an intercostal nerve predominantly provides remaining sensation to the nipple-areola complex?

A) Anterior branch of the fifth
B) Anterior branch of the fourth
C) Lateral branch of the fifth
D) Lateral branch of the fourth
E) Lateral branch of the third

A

The correct response is Option B.

The cutaneous innervation of the female breast is derived medially from the anterior cutaneous branches of the first to sixth intercostal nerves and laterally from the lateral cutaneous branches of the second to seventh intercostal nerves. The nipple-areola complex is physiologically innervated by the lateral and anterior branches of the third to fifth intercostal nerves. The fourth intercostal nerve has further shown to be most consistent in various anatomical studies.

However, the anterior branches take a superficial course within the subcutaneous tissues of the medial breast while the lateral branches take a deep course within the pectoral fascia and reach the nipple via the breast parenchyma. Therefore, the lateral branches are most likely resected during mastectomy and contribute little to the postoperative innervation of the nipple-areola complex.

How well did you know this?
1
Not at all
2
3
4
5
Perfectly
37
Q

A 65-year-old woman with a history of left mastectomy for breast cancer 10 years ago undergoes biopsy of a suspicious lesion of the right breast found on a recent mammogram. Examination of the biopsy specimen confirms a right breast carcinoma. This lesion most likely originated from which of the following structures?

A) Adipose tissue
B) Areolar skin
C) Lactiferous duct
D) Lymph node
E) Pectoralis major muscle

A

The correct response is Option C.

Women who were previously treated for breast cancer are at increased risk for development of a metachronous lesion of the contralateral breast. Cancers of the breast are typically adenocarcinomas, arising from the glandular tissue such as the ducts or lobules. Paget disease of the breast would involve the areolar skin but is fairly uncommon. Sarcomas arising from the connective tissue (such as adipose or muscle) are also rare. Breast adenocarcinomas do not originate from lymphatic tissue.

How well did you know this?
1
Not at all
2
3
4
5
Perfectly
38
Q

A 46-year-old woman with cancer of the right breast comes to the office to discuss a right mastectomy with immediate implant reconstruction and a symmetry procedure for the contralateral breast. Physical examination shows bilateral Grade 3 ptosis with volume symmetry. The patient currently wears a size 34C brassiere and desires to remain the same size. Which of the following is the most appropriate procedure for the contralateral breast?

A) Augmentation mammaplasty
B) Fat injections
C) Mastopexy
D) Reduction mammaplasty

A

The correct response is Option C.

Contralateral breast procedures are frequently necessary to achieve symmetry following mastectomy and reconstruction. Options to achieve symmetry include mastopexy, reduction mammaplasty, or augmentation mammaplasty combined with mastopexy. Volume symmetry can be achieved through reduction mammaplasty or fat injections. Positional asymmetry of the contralateral breast and the nipple-areola complex may require correction with mastopexy. Augmentation mammaplasty with mastopexy may be indicated for the correction of volume and positional asymmetry. The most appropriate contralateral procedure in a patient with symmetric volumes of the breast and ptosis of the contralateral breast is mastopexy.

How well did you know this?
1
Not at all
2
3
4
5
Perfectly
39
Q

Which of the following is most commonly associated with alloplastic breast reconstruction in the setting of adjuvant radiation therapy?

A) Decreased rates of implant rupture
B) Decreased rates of seroma
C) Increased patient satisfaction
D) Increased risk for capsular contracture

A

The correct response is Option D.

Alloplastic breast reconstruction increases the risk for capsular contracture in the setting of adjuvant radiation therapy. It can also increase the risks for seroma, wound healing complications, and infections. Radiation would not decrease the rates of implant rupture and would not increase patient satisfaction.

How well did you know this?
1
Not at all
2
3
4
5
Perfectly
40
Q

A 36-year-old woman with invasive ductal carcinoma of the left breast comes to the office to discuss options for immediate breast reconstruction. Examination shows that the tumor is positive for estrogen receptor (ER) and progesterone receptor (PR) expression. Family history is negative for breast and ovarian cancer, and genetic testing shows no abnormalities. Bilateral mastectomy is planned. Which of the following outcomes is most likely in this patient following contralateral prophylactic mastectomy of the right breast?

A) Decreased number of anticipated overall operations
B) Decreased risk of recurrence of the known cancer
C) Increased cure rate for her known cancer
D) Increased possibility of future breast-feeding
E) Increased risk of surgical site complications

A

The correct response is Option E.

Contralateral prophylactic mastectomy (CPM) has become increasingly common in recent years. Women who are at higher than average risk of developing breast cancer may have a clear oncologic benefit from CPM in terms of risk reduction. This would include women who are carriers of high-risk genetic mutations such as BRCA1 and BRCA2, women with a strong family history of breast cancer, and young women with high risk of aggressive tumors. However, for women with average risk of contralateral breast cancer (2 to 6% over 10 years), studies have not supported an oncologic benefit to CPM, attitudes of the lay public and media attention notwithstanding.

Since the surgeries of each breast, involving both mastectomy and reconstruction, are largely (although not completely) independent of one another, the risk of surgical complications to one side or the other is nearly additive, resulting in an approximate doubling of those risks.

CPM has not been shown to improve the cure rate for the known cancer treated with therapeutic mastectomy.

CPM has not been shown to reduce risk of recurrence for the known cancer on the primary side.

CPM would not be expected to decrease the number of anticipated operations. Given the increased likelihood of complications, one may reasonably expect the same number of operations or greater (for treatment of complications), but not fewer.

Removal of both breasts eliminates the ability to breast-feed in the future.

How well did you know this?
1
Not at all
2
3
4
5
Perfectly
41
Q

A 75-year-old woman is evaluated after undergoing bilateral autologous breast reconstruction with deep inferior epigastric artery perforator (DIEP) flaps 2 days ago. The patient had been recovering well until this morning, when she developed acute respiratory insufficiency with hypoxia, tachycardia, and tachypnea. Pulmonary embolism is suspected, and pulmonary CT angiography has been ordered. Baseline creatinine level is 1.1 mg/dL. Which of the following steps should be taken to prevent contrast-induced nephropathy in this patient?

A) Hydration with hydroxyethyl starch (colloid)
B) Hydration with 0.9% saline
C) No preventive measures are needed
D) Pretreatment with the antioxidant N-acetylcysteine
E) Pretreatment with 0.9% saline and diphenhydramine

A

The correct response is Option B.

Although the risk of contrast-induced nephropathy (CIN) is overall relatively low, geriatric patients are at higher risk for this complication secondary to their high prevalence of risk factors for renal dysfunction. In this patient population, even normal laboratory creatinine concentrations are not always indicative of normal glomerular filtration rate (GFR), and every effort should be made to prevent the development of CIN. Important prophylactic strategies to avoid this adverse effect include optimization of fluids and avoidance of dehydration, use of non-ionic contrast media, and avoiding repeated exposure to contrast at close intervals.

This geriatric patient should be well hydrated prior to undergoing exposure to this high-contrast study. Minimizing fluid and performing no intervention will increase the patient’s risk of developing CIN even with a normal creatinine concentration. Antioxidants such as N-acetylcysteine, vasodilators such as dopamine, and colloid solutions such as Hespan have not proven to be beneficial in preventing renal dysfunction. Pretreatment with 0.9% saline and diphenhydramine is used in patients who have allergies to the contrast dye to minimize the allergic reaction and plays no role in preventing CIN.

42
Q

Which of the following factors is most likely to lead to a patient undergoing breast reconstruction after mastectomy?

A) BRCA1/BRCA2 status
B) Insurance coverage through Medicaid
C) Living close to a reconstructive surgeon
D) Patient age
E) Residence in the southeastern United States

A

The correct response is Option C.

Several large population-based studies in different regions of the United States show that provision of breast reconstruction is not uniform among all mastectomy patients. The two biggest factors are geographic distance from reconstructive surgeons and insurance status. Patients who live farther away from providers of reconstruction and those uninsured or with Medicaid have a much lower incidence of reconstruction compared to the overall cohorts. Most studies also show BRCA positive and younger patients tend to have a higher incidence of bilateral reconstruction.

43
Q

Following a skin-sparing mastectomy, a 39-year-old woman undergoes deep inferior epigastric perforator (DIEP) flap breast reconstruction. To augment flap sensation, the anterior sensory branch of the fourth intercostal nerve is coapted to which of the following nerves within the DIEP flap?

A) Genitofemoral
B) Iliohypogastric
C) Ilioinguinal
D) Intercostal
E) Lateral femoral cutaneous

A

The correct response is Option D.

The third, fourth, and fifth intercostal nerves are responsible for innervation of the majority of the breast. The anterior branch of the fourth intercostal nerve provides most erogenous sensation to the nipple. Sensation to the lower abdomen arises from segmental cutaneous branches of the intercostal nerve, which travel through the rectus abdominis muscle. T10 provides sensation to the dermatome, including the periumbilical region, and is most commonly used. The iliohypogastric nerve provides sensation to the lateral gluteal region. The ilioinguinal nerve provides sensation to the upper medial thigh. The genitofemoral nerve provides sensation to the upper anterior thigh and mons pubis. The lateral femoral cutaneous nerve provides innervation to the lateral thigh and is not used for this purpose.

44
Q

Which of the following is a risk factor for hormone-sensitive breast cancer?

A) Breast-feeding
B) Early age at first pregnancy
C) Early menopause
D) Late menarche
E) Post-menopausal obesity

A

The correct response is Option E.

Post-menopausal obesity is associated with increased adipose production of estrogen, which can increase the risk for hormone-sensitive breast cancer. Other options (late menarche, early menopause, and breast feeding) decrease the number of menstrual cycles, and therefore may decrease the risk of breast cancer. Early age at first pregnancy is also associated with decreased risk for hormone-sensitive breast cancers.

45
Q

A 54-year-old woman with BRCA mutation is scheduled for a bilateral nipple-sparing mastectomy. Which of the following would lead to the highest risk for postoperative nipple necrosis?

A) Grade 2 breast ptosis
B) Periareolar mastectomy incision
C) Previous excisional breast scar
D) Tumor greater than 1.5 cm
E) Use of smooth round gel implants

A

The correct response is Option B.

Nipple-sparing mastectomy and direct to implant reconstruction is becoming increasingly popular in the setting of prophylactic mastectomies. The criteria for nipple-sparing mastectomies have been increased to tumors not involving the nipple-areola complex, with some surgeons requiring a distance of 2 cm from the nipple and others espousing just a clean surgical margin at the nipple. Nipple-sparing mastectomies do not require recreating the breast envelope and therefore direct to implant reconstruction is possible. Increased risk for incidence of nipple necrosis in the setting of direct to implant reconstruction is associated with mastectomy incision involving the areola. The use of smooth round gel implants, grade 2 ptosis, tumor size, or previous breast biopsy scars are not associated with nipple necrosis.

46
Q

An otherwise healthy 16-year-old girl comes to the office because of a painless mass in the left breast. Physical examination of the left breast discloses a circumscribed firm, rubbery, 3-cm mass without overlying skin changes, and no axillary lymphadenopathy. Results of a pregnancy test are negative. Which of the following is the most likely diagnosis?

A) Common fibroadenoma
B) Giant fibroadenoma
C) Lactating adenoma
D) Phyllodes tumor
E) Tubular adenoma

A

The correct response is Option A.

A common fibroadenoma is the most likely diagnosis of this patient. Common fibroadenoma is the most common breast tumor in adolescent females and present between the ages of 14 and 16.

Juvenile fibroadenoma is a variant of fibroadenoma and is usually seen in adolescents and young adults. It is associated with a normal stromal/epithelial balance, which distinguishes it from phyllodes tumor, and has both stromal and epithelial hyperplasia. In addition, juvenile fibroadenomas are characterized by rapid growth.

A giant fibroadenoma is a clinical diagnosis, rather than a pathologic diagnosis. It is characterized by its size, usually greater than 5 cm.

Complex fibroadenoma is characterized by fibrocystic changes on glandular tissue with underlying features of common fibroadenoma on pathologic analysis.

A tubular adenoma has glandular proliferation on pathologic analysis, and while it is a subtype of fibroadenoma, it is not as common as common fibroadenoma. A lactating adenoma, similarly, is defined by the presence of secretory hyperplasia of lobules on pathologic analysis. Lactating adenomas are so defined because of the histologic presence of secretory hyperplasia, and they commonly occur in pregnant or lactating women. Many lactating adenomas will spontaneously regress.

A phyllodes tumor is typically a large, rapidly growing lesion and can be either benign or malignant. It is rare in adolescents, but when found, is usually aggressive. Phyllodes tumor is related to fibroadenoma and is distinct from other forms of breast cancer. Treatment is wide local excision.

47
Q

A 30-year-old woman comes to the office because of pain in the left breast. Two weeks ago, she underwent core needle biopsy of a breast mass that was diagnosed as benign. Family history does not include breast cancer. She does not smoke cigarettes. On examination, the left breast is erythematous and tender to palpation, and the skin of the breast is retracted laterally. There is a palpable, rope-like mass that courses longitudinally along the breast. Which of the following is the most appropriate diagnosis and treatment of this lesion?

A) Breast abscess, perform incision and drainage of the mass
B) Fibrocystic changes, perform biopsy
C) Mastodynia, treat with 10-day course of broad-spectrum antibiotics
D) Nipple papilloma, perform diagnostic mammography
E) Superficial thrombophlebitis, manage with analgesics

A

The correct response is Option E.

Mondor disease, or superficial thrombophlebitis of the breast, involves the superficial veins of the breast and anterior chest wall. It may occur following surgery, core biopsy, irradiation, or trauma. Clinical manifestations include pain, redness and swelling, and the presence of a thickened tender cord. This condition usually resolves in 4 to 6 weeks with symptomatic treatment using pain relief.

Nipple papillomas may be identified as a mass on breast imaging or may be found incidentally. They frequently present with bloody nipple discharge. While not concerning in and of themselves, these lesions may harbor areas of atypia or ductal carcinoma in situ, and are treated with core needle biopsy.

While a breast abscess is possible after a diagnostic procedure such as a biopsy, it would present as localized swelling, tenderness, and induration. The skin retraction and rope-like mass would not be present.

Fibrocystic changes in the breast present as a solitary mass and may cause patients to seek medical attention because of associated pain. They may fluctuate in size and tenderness during a patient’s menstrual cycle. Because no breast mass can be definitively declared benign on physical examination alone, imaging and/or biopsy may be considered.

Breast pain in the absence of a finding on physical examination may have a number of causes including menstrual changes, breast hypertrophy, diet, hormone replacement therapy, ductal ectasia, mastitis, malignancy, and hidradenitis. While the patient does have breast pain, the other physical findings rule out mastodynia alone.

48
Q

Oral contraceptive use increases the incidence of which of the following histologic risk factors for breast cancer?

A) Apocrine metaplasia
B) Atypical hyperplasia
C) Intraductal hyperplasia
D) Intraductal papilloma
E) Sclerosing adenosis

A

The correct response is Option B.

Patients in whom an otherwise benign breast biopsy shows atypical hyperplasia have a 4.5- to 5-fold increased risk for developing breast cancer. Proliferative breast disease without atypia increases the risk 1.5- to 2-fold. Oral contraceptives have shown to decrease the occurrence of all proliferative forms of benign breast disease without atypia, such as intraductal hyperplasia, intraductal papilloma, and sclerosing adenosis. Apocrine metaplasia is a non-proliferative histologic change and carries no increased risk for breast cancer. Of the choices, only atypical hyperplasia is increased with oral contraceptive use.

49
Q

A 42-year-old nurse is scheduled to undergo elective non–implant-based surgery of the left breast. Medical history includes no abnormalities, and she has no allergies. She smokes 1 pack of cigarettes daily. To decrease the incidence of surgical site infection, which of the following is the most effective perioperative strategy?

A) Administer cefazolin intravenously within 5 minutes of skin incision
B) Administer an insulin drip to keep blood glucose concentration less than 100 mg/dL
C) Have the patient practice complete smoking cessation for 7 days prior to surgery
D) Prescribe nasal mupirocin and chlorhexidine baths for 5 days prior to surgery
E) Use povidone-iodine skin preparation rather than chlorhexidine/isopropyl alcohol

A

The correct response is Option D.

A 2010 randomized controlled trial of over 6700 patients published in the New England Journal of Medicine documented a nearly 60% decrease in Staphylococcus aureus infections in patients if mupirocin was applied twice a day to the nares as well as a full-body wash with chlorhexidine for 5 days prior to surgery. The mean hospital stay was already reduced almost 2 days. A meta-analysis also demonstrated the same findings, with a nearly 45% decrease in surgical site infections (SSIs).

Data would suggest that isopropyl alcohol–containing skin preparations for surgery decrease SSI rates more effectively than povidone-iodine alone.

Perioperative antibiotics should be administered with enough advance time to achieve proper and adequate rates of skin penetration. With cefazolin, this is 30 to 59 minutes before skin incision, with an odds ratio of 1.0 (vs. 2.0 if given within 30 minutes, and 1.7 if given after 60 minutes). Five minutes prior to skin incision is not sufficient for the SSI-reducing effect to be achieved.

Blood glucose control is critical to decreasing SSI rates, with optimal rates usually being quoted as less than 180 mg/dL. However, this patient is not diabetic; she is otherwise healthy, so an insulin drip would not be appropriate. In addition, hypoglycemia can also have detrimental physiologic effects and also should be avoided.

Smoking cessation decreases SSI rates if the patient does not smoke for 4 weeks before or after surgery. A 2012 systematic review and meta-analysis of nearly 500,000 patients demonstrated this. However, 7 days of smoking cessation is insufficient time in advance of surgery to obtain these statistically significant benefits.

50
Q

A 45-year-old woman comes to the office for consultation regarding severe breast asymmetry after undergoing lumpectomy and radiation therapy for ductal carcinoma of the right breast 5 years ago. Physical examination shows the radiated right breast is tight and retracted, and the left breast is ptotic. Left-sided mastopexy for symmetry and autologous fat grafting to the radiated breast are recommended. The patient asks if the procedure will be covered by insurance. Which of the following is the most appropriate response?

A) Fat grafting the right breast will be covered by insurance but the mastopexy will be considered cosmetic and will not be covered
B) Insurance companies rarely cover the cost of immediate breast reconstruction
C) Insurance may not cover the procedure since insurers are not mandated to pay for reconstruction of lumpectomy defects
D) Procedures for both sides will be covered by insurance
E) The left-sided mastopexy will likely be covered but the fat grafting will not be covered

A

The correct response is Option C.

The Women’s Health and Cancer Rights Act, signed into law in 1998, requires insurance plans to cover the cost of breast reconstruction after mastectomy. The law includes all stages of reconstruction as well as contralateral procedures to provide symmetry. An often misunderstood aspect of the Women’s Health and Cancer Rights Act is that it does not apply to individuals undergoing breast conservation therapy (lumpectomy with radiation). As rates of breast conservation therapy have continued to increase (60% of women with stage I cancers), so have significant lumpectomy defects associated with the untoward effects of radiation. In the clinical scenario presented, the patient should be informed that the corrective operation (fat grafting and mastopexy) might not be covered by her insurance company.

51
Q

A 35-year-old woman comes to the office to discuss a recent diagnosis of breast cancer. Recent mammography showed diffuse microcalcifications throughout the breast, and needle biopsy showed infiltrating ductal carcinoma. On physical examination, some retraction of the skin in the lower outer quadrant of the breast is noted. She wears a size 36C brassiere. The patient reports that she is currently considering whether to have lumpectomy and radiation therapy or mastectomy. Which of the following features of this clinical scenario is a CONTRAINDICATION to breast conservation therapy?

A) Breast size
B) Mammographic findings
C) Patient age
D) Skin retraction on physical examination of the breast
E) Tumor pathology

A

The correct response is Option B.

Breast conservation therapy (BCT) refers to breast conserving therapy followed by moderate-dose radiation to eradicate microscopic residual disease. The goal is to provide the equivalent survival of mastectomy while maintaining a cosmetically acceptable appearance with a low rate of recurrence.

When considering breast conservation therapy or mastectomy, the needs and desires of each patient should be addressed. Age alone is not a contraindication to BCT, but overall health and comorbidities should be considered. Histologic subtype and pathology are not contraindications to BCT as long as the tumor is not diffuse and can be safely excised with negative margins. Similarly, breast size needs to be considered along with tumor size and location, but “small” or “large” breasts are not indications or contraindications. While retraction of the skin, nipple, or breast parenchyma is not an absolute contraindication to BCT, as long as negative margins can be safely removed, the cosmetic impact of their involvement should be considered.

There are few absolute contraindications to BCT, but they include:

Multicentric disease with two or more tumors in separate quadrants of the breast such that they cannot be encompassed in a single excision

Diffuse malignant microcalcifications on mammography

A history of prior radiation in the same breast or chest wall

Pregnancy

Persistently positive margins despite re-excision

52
Q

A 52-year-old woman undergoes immediate unilateral breast reconstruction with a free deep inferior epigastric flap. Near completion of the procedure, the flap skin paddle is noted to have venous congestion. On exploration of the pedicle, the anastomosis between the vena comitans and the internal mammary vessel appears patent. Which of the following preventive measures would most likely have averted this issue?

A) Administration of heparin immediately before release of vessel clamps and flap revascularization
B) Anastomosis of the veins using sutures instead of a venous coupling device
C) Preservation and anastomosis of the superficial inferior epigastric vein
D) Routine anastomosis of two venae comitantes per flap
E) Use of near-infrared fluorescence imaging to assess flap blood flow

A

The correct response is Option C.

Preservation of the superficial inferior epigastric veins (SIEV) during flap harvest is a useful preventive measure in microsurgical free tissue transfer operations. These veins can serve as important lifeboats to augment venous outflow in the setting of venous congestion. Typically, if a free flap demonstrates venous congestion, the inset should be taken down and the pedicle, recipient vessels, and anastomoses interrogated. Simple issues, such as mechanical compression or twisting of the vein should be ruled out. Next, the SIEV should be inspected. If it is engorged, it is likely that the flap is reliant on superficial outflow, and this vein should be connected to a recipient vessel to augment the venous outflow of the flap. Options for recipient veins include an anterograde branch on the pedicle vena comitans, or in a retrograde fashion to the vena comitans that was not used in the initial set of anastomoses.

Near-infrared fluorescence imaging technology can assist with flap design and may be useful for assessing the arterial inflow of the flap, but it has not been shown to correlate with flap loss or venous complications.

The use of one or two veins in microsurgical free tissue transfer is a topic that has been debated for several years. While some studies indicate that the use of two venous connections may decrease the velocity of blood flow across the anatomosis, there are no data to support differences in flap outcomes or thrombotic events. Therefore, the routine use of a second vein is largely up to surgeon preference.

Heparin may be a useful adjunct when thrombosis of the arterial or venous anastomosis is excised and a revision of the anastomosis is performed. Without evidence of thrombosis, it is unlikely to have any added benefit.

For venous anastomoses, the use of venous coupling devices has not been associated with patency issues or increased thrombosis rates. Therefore, a hand-sewn anastomosis is not likely to prevent the issue presented in this question.

53
Q

A 36-year-old woman with genetic susceptibility to breast cancer is scheduled to undergo bilateral prophylactic mastectomy. She has elected to proceed with immediate single-stage reconstruction using permanent silicone implants and acellular dermal matrix. Which of the following is the most sensitive method to detect implant rupture in this patient?

A) CT scan
B) Mammography
C) MRI
D) Physical examination
E) Ultrasonography

A

The correct response is Option C.

Implant rupture is one of the most common reasons for implant removal. While implant rupture can be associated with symptoms such as capsular contracture, it is often completely asymptomatic or “silent.” For this reason, screening imaging to detect such ruptures has been recommended.

According to the 2011 Update on the Safety of Silicone Gel-Filled Breast Implants published by the U.S. Food and Drug Administration, it is currently recommended that women with silicone implants get their first MRI 3 years after they receive the implants and every 2 years thereafter to detect silent ruptures.

Since the rate of rupture increases the longer an implant is in place, not screening is unacceptable.

Mammograms can detect extracapsular silicone when an implant ruptures, but they do not detect intracapsular ruptures.

The accuracy of ultrasound largely depends on the skill of the ultrasound technologist, the type of equipment used, and the experience of the interpreting physician. Furthermore, ultrasound is limited in its ability to detect ruptures in the back wall of the implant and in the breast tissue behind it.

CT scans can detect intracapsular silicone gel-filled breast implant rupture, but they are limited in their ability to detect extracapsular ruptures.

54
Q

A 53-year-old woman comes to discuss breast reconstruction after undergoing left mastectomy for ductal carcinoma in situ. She does not require chemotherapy or radiation therapy and does not want surgery on the unaffected breast. She is obese but otherwise healthy with a large, ptotic right breast. Which of the following breast reconstruction techniques is most likely to result in the greatest long-term patient satisfaction?

A) Autologous breast reconstruction with microvascular free tissue transfer from the abdomen
B) Extended latissimus dorsi musculocutaneous flap reconstruction without an implant
C) Immediate reconstruction with cohesive, anatomically shaped silicone gel implant
D) Immediate tissue expander followed by implant-based reconstruction
E) Latissimus dorsi musculocutaneous flap reconstruction with immediate placement of a permanent implant

A

The correct response is Option A.

Multiple studies have supported improved patient satisfaction with autologous breast reconstruction in the setting of a unilateral reconstruction. The clinical scenario involves an obese patient with a large, ptotic breast who does not desire a surgical procedure for symmetry and is the ideal candidate for an autologous reconstruction. Microvascular free tissue transfer of abdominal tissue for breast reconstruction demonstrates improved reliability and decreased fat necrosis compared with pedicled flap reconstruction. Implant-based reconstruction is less likely to provide adequate symmetry in this patient. An extended latissimus dorsi musculocutaneous flap–based reconstruction alone is unlikely to provide enough volume for symmetry and a traditional latissimus flap with implant is unlikely to provide adequate ptosis for symmetry.

55
Q

A 43-year-old woman would like to discuss plans for breast reconstruction after her upcoming unilateral mastectomy. Postoperative radiation therapy is planned. Which of the following is the most likely benefit of tissue expander–based breast reconstruction compared with immediate autologous breast reconstruction using this patient’s abdominal tissue?

A) Better symmetry
B) Improved postoperative sensation
C) A larger, more ptotic breast reconstruction
D) Lower risk of complications from radiation
E) Preservation of the patient’s options for final reconstruction

A

The correct response is Option E.

Immediate breast reconstruction with tissue expanders followed by reconstruction of choice preserves the patient’s skin envelope and keeps open options for definitive final reconstruction of choice whether with an implant or autologous tissue.

Tissue expander-based reconstruction is associated with a higher complication rate in the setting of radiation therapy but preserves abdominal and back tissue as options for autologous reconstruction. Implant-based reconstruction does not provide the advantages of improved symmetry, sensation, or breast ptosis.

56
Q

A 62-year-old woman undergoes breast reconstruction using autologous tissue from the abdomen. Intraoperatively, use of a perforator flap is found to be impossible because of multiple small nondominant perforators. Conversion to a delayed pedicled transverse rectus abdominis muscle flap is planned. Ligating which of the following vessels in this stage will best facilitate future viability of the tissue transferred in the next stage?

A) Deep inferior epigastric
B) Hypogastric
C) Internal mammary
D) Superficial inferior epigastric
E) Superior epigastric

A

The correct response is Option A.

Pedicled transverse rectus abdominis muscle (TRAM) flaps are based on the superior epigastric system, which is often less robust than the deep inferior epigastric system. Therefore, surgical delay by ligation of the deep inferior epigastric system may facilitate overall viability of the transferred tissue. Ligation of the superior epigastric system would make a pedicled TRAM flap unlikely to survive. The internal mammary ligation may also interrupt blood supply to the superior epigastric system, and even if the tissue is fed through collaterals, it would not strengthen the flap. Division of the superficial inferior epigastric system might also help, but it is not as critical as ligation of the deep inferior epigastric system. The hypogastric system does not have a direct impact on the pedicled TRAM tissues.

57
Q

A 35-year-old BRCA-positive woman undergoes bilateral prophylactic nipple-sparing mastectomy via lateral radial incisions with periareolar extensions. She undergoes direct-to-implant reconstruction with an acellular dermal matrix inferior lateral sling. Mastectomy specimens weigh 435 g each, and placement of 450 mL smooth, round, high-profile implants is performed. Postoperatively, 25% partial nipple areolar complex necrosis in both breasts is noted. Which of the following most likely contributed to the necrosis?

A) Bilateral surgery
B) Mastectomy incision
C) Use of acellular dermal matrix
D) Volume of the implants
E) Weight of the mastectomy specimen

A

The correct response is Option B.

Nipple-sparing mastectomies are becoming more prevalent with the rise of prophylactic mastectomies and increased comfort with performing it in presence of in situ and invasive carcinoma. While universal standards for patient selection do not exist, there is consensus that general recommendations include no inflammatory breast cancers, no pathologic nipple discharge, no Paget’s disease, and for many, a 2-cm distance from the tumor to the nipple. Nipple-sparing mastectomy incisions have been described as radial lateral, lateral inframammary fold, and peri-areolar. Because the entire skin envelope is preserved, direct-to-implant reconstruction is possible with inferolateral support from acellular dermal matrices or other scaffolds. Areolar involvement in mastectomy incisions is associated with increased rates of nipple necrosis. The weight of mastectomy specimen, volume of implants, use of acellular dermal matrices or laterality of surgery have not been shown to be associated with nipple necrosis.

58
Q

An otherwise healthy 38-year-old woman who is a smoker is considering implant-based breast reconstruction following mastectomy. She has been counseled about likely use of acellular dermal matrix when the intermediate tissue expander is placed and wants to further understand why the matrix will be used in her body. The patient should be advised that use of acellular dermal matrix is associated with a decreased risk for which of the following?

A) Cancer recurrence
B) Capsular contracture
C) Seroma
D) Skin flap necrosis
E) Smoking-related complications

A

The correct response is Option B.

Acellular dermal matrix (ADM) use has been increasing over time with tissue expander or implant-based breast reconstruction following mastectomies. Many potential advantages and disadvantages have been studied, and some of the data are contradictory. However, the consensus of the literature indicates that ADMs are associated with decreased capsular contracture rates. There is literature to suggest that seroma rates are increased or remain stable, not decreased, with ADMs. ADMs have not been shown to decrease independent patient risk factors for complications such as tobacco use or to decrease cancer recurrence rates. ADMs also do not appear to improve vascularity of the tissue overlying them when initially placed.

59
Q

A 45-year-old woman undergoes mastectomy. Which of the following anatomical landmarks denotes a limit of the breast and, therefore, a limit to the extent of the mastectomy?

A) Anterior border of latissimus dorsi muscle
B) Inferior origin of pectoralis major muscle
C) Lateral pectoralis minor muscle
D) Superficial fascia of the serratus anterior muscle
E) Supraclavicular lymph node basin

A

The correct response is Option A.

The borders of the breast and, therefore, the limits of the mastectomy are the sternum, clavicle, inframammary fold, and anterior border of the latissimus dorsi muscle. The other options are not designated as borders of the breast tissue.

60
Q

A 57-year-old woman with a history of modified left radical mastectomy followed by adjuvant chemotherapy and radiation therapy is evaluated six months after her last radiation treatment for breast reconstruction. Physical examination shows slight skin redundancy of the chest wall and grade 2 ptosis of the contralateral breast. She wears a size 36D brassiere and does not want to undergo symmetry procedures on the contralateral breast. Which of the following reconstructive options is most likely to achieve breast symmetry in this patient?

A) Deep inferior epigastric perforator flap
B) Latissimus dorsi flap
C) Thoracodorsal artery perforator flap
D) Tissue expander with acellular dermal matrix and staged implant placement
E) Total submuscular tissue expander and staged implant placement

A

The correct response is Option A.

Breast reconstruction requires reconstruction of both the soft-tissue envelope and volume. In the setting of radiated chest wall tissues, it would be difficult to recreate a breast envelope of sufficient size to match the contralateral breast with tissue expansion, even with acellular dermal matrix or total submuscular placement of a tissue expander. The history of radiation would likely cause an element of capsular contracture and decrease symmetry to the contralateral breast with grade 2 ptosis. Both the latissimus dorsi flap and the thoracodorsal artery perforator flap would recruit non-radiated skin to create an adequate envelope, but would not provide adequate volume alone. The deep inferior epigastric perforator (DIEP) flap would provide both envelope and volume.

61
Q

A 53-year-old woman with a BMI of 27 kg/m2, gigantomastia, and grade III ptosis is considering unilateral mastectomy and autologous reconstruction with abdominal tissue. Which of the following is the most appropriate advice regarding the contralateral breast?

A) A contralateral prophylactic mastectomy should be performed to maximize symmetry
B) Contralateral reduction should be accompanied by placement of an implant on that side to address upper pole symmetry with the autologous reconstruction
C) Simultaneous contralateral reduction may be performed with an acceptable risk profile
D) Symmetry is unlikely to be achieved, so a contralateral matching procedure would not be recommended
E) Symmetry with autologous tissue is likely to be achieved without the contralateral matching procedure

A

The correct response is Option C.

Studies have shown that contralateral symmetry procedures performed synchronously with unilateral autologous tissue reconstruction after mastectomy (including reconstructions with a free perforator flap) are acceptable. Situations requiring reduction rather than augmentation or mastopexy seem to be the most suited to this timing. A patient with a lower BMI and gigantomastia is unlikely to achieve symmetry without a contralateral procedure. Autologous tissue reconstruction likely would not need a contralateral implant for upper pole symmetry. An attempt at improvement in symmetry using contralateral surgery would be acceptable should the patient so choose, but recommending a prophylactic mastectomy on the contralateral side solely for symmetry and not for risk reduction may be overly aggressive when other methods such as reduction mammaplasty exist with a likelihood of acceptable postoperative symmetry.

62
Q

Which of the following is the most accurate location of the elliptical skin island of a profunda artery perforator (PAP) flap?

A) Anteromedial thigh with the superior border within the groin crease
B) Inferior buttock with the inferior border within the gluteal fold
C) Lateral hip superior to the iliac crest
D) Middle buttock, from the posterior superior iliac spine to the apex of the greater trochanter
E) Posteromedial thigh with the superior border within the gluteal fold

A

The correct response is Option E.

According to the literature, the skin island of the profunda artery perforator (PAP) flap is inferior to the gluteal fold.1,2 An ellipse of the inferior buttock with the inferior border within the gluteal fold describes the skin island of the inferior gluteal artery perforator (IGAP) free flap.3 An ellipse of the anteromedial thigh with the superior border within the gluteal fold describes the transverse upper gracilis (TUG) flap. An ellipse of the middle buttock, from the posterior superior iliac spine to the apex of the greater trochanter, describes the superior gluteal artery perforator (SGAP) flap.3 An ellipse of the lateral hip superior to the iliac crest describes the Rubens or lateral hip flap.4 The only option that correctly identifies the skin island for the PAP flap is an ellipse of the posteromedial thigh with the superior border within the gluteal fold. The superior marking is within or just below the gluteal fold and the inferior marking is roughly 7 cm below the superior marking. The flap is an ellipse so the scar does not extend outside of the gluteal fold.1,2

63
Q

Nipple-sparing mastectomy is CONTRAINDICATED in which of the following women considering mastectomy and immediate breast reconstruction?

A) A 21-year-old with BRCA1 mutation and grade I breast ptosis
B) A 31-year-old with BRCA2 mutation; subglandular augmentation mammaplasty 1 year ago
C) A 45-year-old with unilateral breast cancer and need for postoperative radiation therapy
D) A 50-year-old with unilateral breast cancer in the tail of Spence
E) A 61-year-old with unilateral breast cancer; tumor-to-nipple distance of 1 cm

A

The correct response is Option E.

Nipple-sparing mastectomy is increasingly prevalent owing to perceived improvement in reconstructive outcome and patient satisfaction. Prevalence of nipple ischemia, which can vary in severity from incomplete, partial-thickness epidermolysis to total nipple loss, ranges from 2 to 60% depending on the definition used. A recent pooled analysis found a cumulative prevalence of 7%. Most cases respond to local wound care; reoperation for total nipple loss is relatively infrequent.

Patient selection remains paramount to successful nipple preservation. Typically, thin, non-smoking patients with small, non-ptotic breasts are considered ideal candidates. Severe macromastia and ptosis may not only increase risk of poor nipple vascularity, but also contribute to nipple malposition. Oncologic determinants of the safety of nipple preservation should also be considered. Noninflammatory cancers located in excess of 2 cm from the nipple can generally be safely extirpated without removal of the nipple. Usually, an intraoperative frozen section biopsy of the retroareolar tissue (“doughnut”) is performed after excision of the main mastectomy specimen to demonstrate nipple margins free of tumor. Patients undergoing prophylactic mastectomy in the setting of high genetic predisposition are also considered good candidates. Successful nipple preservation has also been described in patients with a history of reduction mammaplasty or mastopexy.

Successful nipple-sparing mastectomy with implant reconstruction has been described recently in patients who require postoperative radiation, albeit with higher risk of capsular contracture and nipple malposition. Prior augmentation in the subglandular or subpectoral positions does not contraindicate nipple-sparing mastectomy; rather, the latter group may present opportunities for direct-to-implant reconstruction. Location of a breast cancer in the tail of Spence (axillary extension of breast tissue) should not contraindicate nipple-sparing mastectomy, as it is likely to be far enough from the nipple to safely spare it.

64
Q

A 62-year-old woman with a history of Stage III breast cancer is scheduled for delayed autologous breast reconstruction from the abdominal donor site. She has no other medical problems. BMI is 30 kg/m2. Her mother had a lower extremity deep venous thrombosis in the past. Caprini risk assessment score is 9. Which of the following is the most appropriate method of postoperative VTE risk reduction?

A) Aspirin therapy
B) Early ambulation after surgery
C) Low-molecular-weight heparin therapy
D) Sequential compression device use
E) No VTE prevention is indicated

A

The correct response is Option C.

Venous thromboembolism (VTE) is a disorder with short-term mortality and long-term morbidity. Plastic and reconstructive surgery patients are known to be at high risk for VTE after surgery. Symptomatic VTE occurs with high frequency after post-bariatric body contouring (7.7%), abdominoplasty (5%), and breast or upper body contouring (2.9%). To fully identify VTE risk in surgical patients, individualized patient assessment is advocated. The Caprini risk assessment model (RAM) is a useful and effective tool to stratify surgical patients for VTE risk. For patients with high Caprini scores, a significantly greater likelihood of VTE events is observed. Approximately 11% of patients with Caprini score >8 will have a VTE within 60 days after surgery.

Based upon recommendations from the ASPS VTE Task Force, patients undergoing elective plastic and reconstructive surgical procedures who have Caprini RAM score of 7 or more should have VTE risk reduction strategies employed, such as limiting operating room times, weight reduction, discontinuation of hormone replacement therapy, and early postoperative mobilization. Patients undergoing major plastic and reconstructive operative procedures performed during general anesthesia that last longer than 60 minutes should receive VTE prevention. For patients with Caprini score of 3 to 6, the use of postoperative low-molecular-weight heparin (LMWH) or unfractionated heparin (UH) should be considered. For patients with Caprini score of 3 or more, use of mechanical prophylaxis throughout the duration of chemical prophylaxis for non-ambulatory patients should be considered. For patients with Caprini score of 7 or more, the use of extended LMWH postoperative prophylaxis should be strongly considered.

Aspirin does not decrease the risk of VTE and may increase the risk of perioperative complications.

65
Q

A 55-year-old woman comes to the office for a second opinion because she is displeased with the results of a recent bilateral mastectomy and breast reconstruction with 800-mL high-profile silicone implants. A photograph is shown. BMI is 35 kg/m2. She repeatedly shows pictures of models with augmented breasts and says that she wants her breasts to be “perkier.” She requests augmentation/mastopexy. Which of the following is the most appropriate next step in management?

A) Augmentation/mastopexy
B) Implant exchange
C) Mastopexy
D) Reassurance
E) Referral to a psychiatrist

A

The correct response is Option D.

The most reasonable approach in this patient is to offer reassurance and reset her expectations. A patient with a BMI of 35 kg/m2 who undergoes mastectomy and implant reconstruction will never look like a model with augmented breasts. This patient clearly has misguided expectations. Any surgical intervention is unlikely to produce the result she is looking for, when in fact she has a very acceptable result as is. Referral of this patient to a psychiatrist will likely upset the patient and undermine her trust.

66
Q

A 42-year-old woman comes to the office for treatment after receiving a diagnosis of cancer of the right breast. She has decided to undergo mastectomy of the right breast. Which of the following is a relative CONTRAINDICATION to nipple-sparing mastectomy?

A) Comedo-type breast tumor
B) Invasive lobular carcinoma
C) Subareolar tumor
D) Tumor location 3 cm from the nipple
E) Tumor size of 2.5 cm

A

The correct response is Option C.

A relative contraindication to nipple-sparing mastectomy is a centrally located tumor. Although various authors have employed different distance criteria, it is generally accepted that patients whose tumors are within 2 cm of the nipple are not candidates for nipple-sparing mastectomy.

Nipple-sparing mastectomy is an appropriate option for high-risk patients undergoing prophylactic mastectomy and for patients diagnosed with breast cancer who meet certain criteria. Those criteria are: tumor size of 3 cm or less, at least 2 cm from the nipple, not multicentric, and with clinically negative nodes.

Comedo carcinoma of the breast is a type of ductal carcinoma in situ. It is considered to be an early stage of breast cancer, is confined to the ducts, and usually does not spread beyond. It is not a contraindication to nipple-sparing mastectomy.

Invasive lobular carcinoma originates from the breast lobules, may form a thickening of the breast tissue rather than a discrete mass, and is often bilateral. As long as it meets the above criteria, it is not a contraindication to nipple-sparing mastectomy.

Inflammatory breast cancer, Paget disease, and tumors infiltrating the skin are also not candidates for skin-sparing or nipple-sparing mastectomy, according to several authors.

In more recent studies, a tumor size of 3 cm or less appears to result in no increase in local or regional recurrence in nipple-sparing mastectomy compared with alternative surgical approaches. A tumor of 2.5 cm is not a contraindication to nipple-sparing mastectomy.

67
Q

Which of the following characteristics is correlated with increased risk of nipple-areola complex necrosis in nipple-sparing mastectomies with immediate reconstruction?

A) Autologous tissue reconstruction
B) Direct to implant reconstruction
C) Patient age
D) Periareolar incision
E) Small breast size

A

The correct response is Option D.

Nipple-sparing mastectomies (NSMs) are becoming more common for both therapeutic and prophylactic mastectomies. Nipple-areola complex (NAC) necrosis can imperil reconstructive efforts, as well as negatively affect patients emotionally. It is important to maximize perfusion to the mastectomy skin flaps and NAC while still performing an oncologically sound procedure. There are multiple different incisions for performing NSM. Periareolar, inframammary-fold, radial, and vertical incisions are the most common. Periareolar incisions are associated with an increased risk of NAC necrosis in NSMs. Type of reconstruction, small breast size, and patient age have not been shown to be linked to increased rates of NAC necrosis.

68
Q

A 52-year-old woman receives a diagnosis of invasive ductal carcinoma of the right breast. Which of the following details from this patient’s history is the strongest risk factor for this diagnosis?

A) Early first pregnancy (less than 30 years)
B) Early menarche (less than 12 years)
C) Early menopause (less than 55 years)
D) Multiple episodes of breast-feeding
E) Remote oral contraceptive use

A

The correct response is Option B.

Early menarche is the highest risk factor for breast cancer of the options listed. Late first pregnancy, late menopause, no breast-feeding, and recent oral contraceptive use are also risk factors for breast cancer but are not as high risk.

69
Q

A 42-year-old woman with a 3-cm invasive ductal carcinoma of the right breast is evaluated for breast reconstruction. She has not decided how she wants to manage her contralateral breast. Regarding eliciting a family history, which of the following cancers is associated with a mutation in a breast cancer-susceptibility gene?

A) Colon
B) Esophageal
C) Lung
D) Pancreatic
E) Thyroid

A

The correct response is Option D.

The breast cancer-susceptibility gene types 1 and 2 (BRCA1 and BRCA2) are tumor suppressor genes. Mutations in BRCA1 and BRCA2 are associated with hereditary breast and ovarian cancers. Additionally, they can be associated with increased risks of pancreatic and prostate cancer. Thyroid, lung, esophageal, and colon cancer are not associated with increased risks of BRCA1 and BRCA2 mutations.

70
Q

A 35-year-old woman comes to the office for consultation regarding prophylactic mastectomy and breast reconstruction. The patient’s mother and sister were diagnosed with bilateral breast cancer in their premenopausal years. Genetic testing for BRCA mutations is negative. Which of the following best estimates this patient’s lifetime risk of breast cancer?

A) 5%
B) 13%
C) 20%
D) 45%
E) 80%

A

The correct response is Option D.

The cumulative lifetime risk for a 35-year-old woman whose mother and sister had breast cancer is estimated to be approximately 15%. The risk may increase to as high as 45% if those cancers were premenopausal and bilateral. BRCA hereditary cancer is characterized by autosomal dominant genetics with multiple family members in each generation being affected. For patients with BRCA1 mutation, the risk of breast cancer has been estimated to be between 50 and 80% by age 65 years. The risk of developing ovarian cancer has been estimated to be 10% by age 60 years.

71
Q

A 39-year-old woman with a history of fibrocystic breast lesions comes to the office for consultation. She has no family history of breast cancer. Results of routine mammograms have been negative; she has never undergone biopsy. Which of the following is the most appropriate recommendation for this patient regarding managing her risk of breast cancer?

A) Continue to schedule routine mammograms
B) Refer for mastectomy
C) Schedule core needle biopsy
D) Schedule fine-needle aspiration
E) Schedule genetic testing

A

The correct response is Option A.

Studies have shown the fibrocystic changes alone in the breast are not directly linked to an increased risk of breast cancer, so there is no indication for fine-needle aspiration or core biopsy. Cancer risk increases in benign breast disease with increased proliferation and atypical hyperplasia. Even with no family history of breast cancer, it is recommended that the patient continue routine mammograms. Fibrocystic breast disease has not been linked in the literature to an increased risk of mutations of the BRCA genes; therefore, genetic testing is unnecessary.

72
Q

An otherwise healthy 37-year-old woman presents for delayed microsurgical breast reconstruction. Which of the following is associated with use of tamoxifen?

A) Hemodynamic instability
B) Impaired wound healing
C) Increased bleeding
D) Seroma formation
E) Thromboembolic events

A

The correct response is Option E.

Breast cancers that are estrogen receptor positive may be responsive to adjuvant chemotherapy with selective estrogen receptor modulators such as tamoxifen, which can reduce recurrence and mortality. Tamoxifen is associated with thromboembolic events, such as deep venous thrombosis and pulmonary embolism. This prothrombotic effect has been postulated to be secondary to the effect of tamoxifen on estrogen receptors that are abundant within vascular endothelium.

Tamoxifen has been shown to be associated with increased rates of total flap loss and decreased rates of flap salvage when taken within 28 days of microsurgical breast reconstruction, which represents two half-lives of the active metabolite of tamoxifen (N-desmethyl tamoxifen, t1/2=14 days). It has therefore been recommended that in patients undergoing microsurgical breast reconstruction, tamoxifen be held for at least 28 days preoperatively. Some authors have further advised holding the medication postoperatively in addition to preoperatively.

Tamoxifen is not associated with impaired wound healing, increased bleeding, hemodynamic instability, or seroma formation.

73
Q

An otherwise healthy 38-year-old woman undergoes prophylactic bilateral mastectomy and immediate reconstruction with deep inferior epigastric artery perforator (DIEP) free flaps. Intraoperatively, the left DIEP flap appears congested before the conclusion of the case. The left deep inferior epigastric artery and vein (DIEA and DIEV) were anastomosed to the proximal internal mammary vessels. The vascular pedicle is evaluated and each anastomosis appears patent and not kinked; however, the venous congestion persists. Which of the following is the most appropriate management?

A) Anastomose the superficial inferior epigastric vein to an internal mammary vessel perforator
B) Convert to left prosthetic reconstruction
C) Infuse tissue plasminogen activator (tPA) to the DIEA
D) Initiate leech therapy
E) Revise the DIEV anastomosis to the retrograde internal mammary vessel limb

A

The correct response is Option A.

Venous drainage of the lower abdominal skin and subcutaneous tissue occurs primarily through the superficial venous system and secondarily through the deep venous system, with perforating veins interconnecting the two systems. These communicating veins have been identified on computed tomography angiography in approximately 90% of abdominal walls in vivo. The majority of the remaining 10% of patients likely have communicating veins that are too small to visualize or are absent. In these cases of anatomical superficial venous system dominance, venous drainage is dependent on the superficial venous system. A recently published 2012 article by Sbitany et al. demonstrated that the incidence of intraoperative venous congestion secondary to persistent superficial venous system dominance was 0.9% in 1201 muscle-sparing transverse rectus abdominis musculocutaneous and deep inferior epigastric artery perforator free flaps. A free flap that becomes congested after reperfusion in the operating room should be assessed immediately for possible etiologies including twisting, kinking, tension, or vasospasm of the vascular pedicle. If a technical problem is ruled out and the venous anastomosis remains patent, obligatory enhancement of venous drainage with the superficial venous system is necessary to salvage the free flap rather than revision of the original anastamosis. Various methods include an anastomosis of the superficial inferior epigastric vein (SIEV) to the DIEV system or any chest wall vein, including the retrograde limb of the internal mammary vessel, the branch of the internal mammary vessel, or the thoracodorsal system. This requires preemptive planning and sparing of the superficial epigastric vein or SIEV during the dissection of the flap. A vein graft can be utilized if additional length is necessary. Another option is to substitute the DIEV anastomosis with the SIEV.

Tissue plasminogen activator (tPA) would not be indicated in this scenario, as it is used as a thrombolytic and there is no evidence of vascular thrombosis. Revising the DIEV anastomosis would be moot because it is patent and the deep system is being drained. Leech therapy is useful for venous congestion, but primarily as an adjunct after potential surgical etiologies have been addressed. Sacrificing the free flap without first attempting salvage is not warranted, and using a prosthetic would be possible only if prior patient consent were obtained

74
Q

A 60-year-old woman with breast cancer undergoes a transverse rectus abdominis musculocutaneous flap breast reconstruction after mastectomy. She has no allergies. Weight is 200 lb (91 kg). Estimated blood loss is 200 mL. Duration of the operation is 3 hours and 50 minutes. Administration of cefazolin before skin incision is planned as prophylaxis against surgical site infection. Which of the following is the most appropriate dosage and timing of this injection?Dose/Timing/Redosing

A) 1g/5 minutes prior/after 150 mL of blood loss

B) 1g/15 minutes prior/no

C)1g/40 minutes prior/no

D)2g/15 minutes prior/no

E)2g/40 minutes prior/no

A

The correct response is Option E.

There has been a renewed interest in perioperative antibiotics in recent years toward more appropriate use to decrease surgical site infection (SSI) while decreasing the incidence of resistant bacteria.

Current recommendations are to administer a single perioperative dose of antibiotics against common skin flora (gram positive), usually using a first-generation cephalosporin. However, the following recommendations may be underappreciated:

Cefazolin intravenous:1 g if <80kg; 2g if >80kg

Alternatives:clindamycin 600 to 900mg intravenously;
vancomycin 1 to 1.5g intravenously

In this case, the patient weighs 200lb (91kg), so 2g of cefazolin is the recommended dosage.

Additionally, an important factor is the timing of the administration of the perioperative antibiotics in order to achieve proper skin levels before incision. In one study by Classen et al., published in the New England Journal of Medicine in 1992, the optimal time was between 2 hours before the operation and skin incision, as greater than 2 hours before and any time after skin incision led to marked increases in the relative risk of SSIs. A follow-up by Weber et al. in 2008 narrowed the most appropriate window to between 30 and 59 minutes before skin incision.

Finally, there is the issue of redosing. Current recommendations are to redose if there is excessive blood loss (>1500 mL) or if there are long procedures where one exceeds the half-life of the antibiotic used.

In the clinical scenario described, the most appropriate choice is 2 g of cefazolin, because the patient’s weight is above 80 kg, administered between 30 to 59 minutes before skin incision. Redosing on the basis of blood loss is unnecessary, although one could consider redosing at approximately 4 hours on the basis of half-life.

75
Q

A 42-year-old woman who underwent mastectomy of the right breast 6 months ago is evaluated for delayed autologous breast reconstruction with free tissue transfer. Which of the following medications should be discontinued preoperatively if she is routinely taking it?

A) Diltiazem
B) Fluoxetine
C) Metoprolol
D) Multivitamin
E) Tamoxifen

A

The correct response is Option E.

It is well known that tamoxifen can increase the risk of thrombembolic events. In a retrospective study at MD Anderson Cancer Center, it was shown that patients who received the drug close to the procedure had a significantly higher rate of complications. It is recommended that the patient stop tamoxifen at least 28 days before surgery.

Other listed medications do not have a direct effect on thrombotic complications.

76
Q

When performing immediate breast reconstruction, it is important to reconstruct the lateral inframammary fold. This is because the oncologic extirpation of the breast is carried out to which of the following anatomic landmarks?

A) Anterior edge of the latissimus dorsi muscle
B) Anterior edge of the serratus muscle
C) Lateral edge of the pectoralis major muscle
D) Lateral edge of the pectoralis minor muscle
E) Posterior edge of the serratus muscle

A

The correct response is Option A.

For modified radical and simple mastectomies, the landmarks of dissection are: superiorly to the clavicle, medially to the sternum, inferiorly to the inframammary fold, and laterally to the border of the latissimus dorsi muscle. The pectoralis major muscle fascia is resected with the specimen.

The recreation of the inframammary fold is important for shaping in breast reconstruction and care must be taken to evaluate and repair both the inferior and lateral components of the inframammary fold.

77
Q

A 52-year-old woman is evaluated for breast reconstruction after modified radical mastectomy with adjuvant chemotherapy and radiation therapy 18 months ago. Her last radiation treatment was 8 months ago. BMI is 29 kg/m2. Examination today shows hyperpigmentation of the right chest wall with no redundancy of the mastectomy skin flaps. Her contralateral breast is a D cup with grade III ptosis. Which of the following methods will create the best symmetry for this patient?

A) Deep inferior epigastric perforator flap
B) Gel breast implant and acellular dermal matrix
C) Latissimus dorsi musculocutaneous flap
D) Tissue expander and acellular dermal matrix placement with planned staged exchange for permanent gel implant
E) Tissue expander placement alone with planned staged exchange for permanent gel implant

A

The correct response is Option A.

The deep inferior epigastric perforator flap would give the patient autologous tissue reconstruction with ample tissue for skin resurfacing and soft tissue for volume. In this radiated patient with a tight skin envelope, a tissue expander/implant, with or without acellular dermal matrix, would be difficult to create an appropriately ample skin envelope and the patient would be at higher risk for wound-healing problems and capsular contracture. The latissimus dorsi musculocutaneous flap, although an autologous tissue reconstruction, would have insufficient volume to adequately match this patient’s contralateral side. It would have to be combined with an implant.

78
Q

An otherwise healthy 67-year-old woman with advanced breast cancer is scheduled to undergo mastectomy and immediate reconstruction with a free transverse rectus abdominis musculocutaneous (TRAM) flap. BMI is 35 kg/m2. Which of the following is most appropriate for deep venous thrombosis prophylaxis?

A) Aspirin and intermittent pneumatic compression stockings
B) Elastic compression stockings only
C) Intermittent pneumatic compression stockings only
D) Low-molecular-weight heparin and intermittent pneumatic compression stockings
E) Positioning and early ambulation

A

The correct response is Option D.

This patient is at highest risk for deep venous thrombosis and she will require combination therapy of compression stockings and chemical prophylaxis.

Perhaps the most well-regarded set of guidelines in this matter comes from the American College of Chest Physicians. These guidelines provide treatment recommendations based on a patient’s risk classification. Caprini model offers a very user-friendly method of calculating patient risk factors and categorizing them into low, moderate, high, and highest risks.

The patient in question has risk factors (age, malignancy, and major surgery) that would put her in the highest risk category. In the highest risk category, prophylaxis will require combination therapy or warfarin.

79
Q

A 48-year-old woman undergoes radical resection of the left breast, left hemisternectomy, four-rib resection, and visceral pleurectomy because of invasive inflammatory breast cancer extending through the thoracic wall into the mediastinum and chest. Examination of specimens obtained from intraoperative frozen section biopsies shows no residual disease. Postoperative adjuvant chemotherapy and radiation therapy to the area are planned. An intraoperative photograph of the 17 × 17-cm defect is shown. Which of the following is the most appropriate technique for soft-tissue reconstruction?

A) Left latissimus muscle flap with skin graft
B) Left rectus muscle turnover flap and skin graft
C) Omental flap and skin graft
D) Reverse abdominoplasty flap
E) Right transverse rectus abdominis musculocutaneous (TRAM) flap

A

The correct response is Option E.

The most appropriate option for this patient would be a right TRAM flap. The defect encompasses a composite defect of the left side of the sternum, including the left internal mammary artery, four contiguous ribs, and the entire left breast. The question specifically asks for soft-tissue thoracic reconstruction. The analysis of the defect yields a very large soft-tissue requirement. Given this, the most logical choice of flaps to provide this amount of soft tissue on a reliable pedicle would be a right (contralateral) TRAM flap. Furthermore, this flap could also be shaped to provide a breast mound versus all the other choices.

A left latissimus muscle flap with skin graft is not the optimal choice as it would not provide sufficient soft-tissue coverage with its associated skin paddle without undue donor site morbidity in this particular patient with a large defect that spans parasternal and lateral chest wall. As a muscle-only flap with skin graft, however, it may be considered as a backup option in case of primary flap failure.

An omental flap with skin graft is an option for reconstruction of this defect; however, it would not be the most appropriate option in this case as it would necessitate an otherwise unnecessary intra-abdominal procedure and would not reshape a breast mound. It has been shown, however, to be optimal in contaminated areas due to its associated lymphatic vessels and nodes associated with its pedicle, so in different circumstances, this may be a preferred choice.

A left rectus turnover flap with skin graft would not be a good option for two reasons: the left internal mammary artery has been harvested, thereby compromising the superior epigastric vessel on which this flap would be based; and it would not cover the entirety of the defect.

A reverse abdominoplasty flap would not be able to reach the full superior extent of the defect.

80
Q

A 45-year-old woman comes to the office for consultation regarding mastectomy and immediate breast reconstruction because of recurrent right-sided breast cancer. She underwent breast-conserving therapy and radiation therapy 5 years ago. BMI is 23 kg/m2. The patient is otherwise healthy, and she does not smoke cigarettes. On examination, the breast is soft with obvious fibrosis. Use of which of the following has the highest risk for reconstructive failure in this patient?

A) Deep inferior epigastric artery perforator (DIEP) flap
B) Free transverse rectus abdominis musculocutaneous (TRAM) flap
C) Latissimus flap with prosthesis
D) Pedicled TRAM flap
E) Tissue expander and prosthesis

A

The correct response is Option E.

Pre-reconstruction radiation is typically seen in two groups of patients: those who underwent mastectomy without reconstruction followed by radiation therapy, or those who underwent breast-conserving therapy and radiation with recurrence or new cancer. Although these patients may present with a reasonable skin envelope, complication rates associated with tissue expander/prosthesis have been reported as high as 70%, with a 40% rate of failure or conversion to flap. Additionally, aesthetic outcomes in most patients who completed expander/prosthesis reconstruction after previous radiation therapy were deemed good or very good compared with the majority of non-irradiated patients who achieved excellent results. Tissue expander/prosthesis can be considered as an option for patients with a history of previous irradiation who wish to avoid the scars and recovery of flap-type reconstructions; however, they should be counseled of the high risks of complications.

The correct response is Option E.

Pre-reconstruction radiation is typically seen in two groups of patients: those who underwent mastectomy without reconstruction followed by radiation therapy, or those who underwent breast-conserving therapy and radiation with recurrence or new cancer. Although these patients may present with a reasonable skin envelope, complication rates associated with tissue expander/prosthesis have been reported as high as 70%, with a 40% rate of failure or conversion to flap. Additionally, aesthetic outcomes in most patients who completed expander/prosthesis reconstruction after previous radiation therapy were deemed good or very good compared with the majority of non-irradiated patients who achieved excellent results. Tissue expander/prosthesis can be considered as an option for patients with a history of previous irradiation who wish to avoid the scars and recovery of flap-type reconstructions; however, they should be counseled of the high risks of complications.

81
Q

A 74-year-old woman comes to the office because of a 2-month history of increasing soreness over the left lateral chest wall. History includes bilateral radical mastectomy and adjuvant radiation to the left chest 30 years ago because of left-sided breast cancer. On examination, a small, nonhealing 1 × 2-cm wound is visible in the subaxillary region within an area of dense fibrotic skin. A photograph is shown. Which of the following is the most appropriate next step?

A) Biopsy of the nonhealing wound
B) Coverage with a left latissimus muscle flap and skin grafting
C) LHyperbaric oxygen treatment
D) MRI of the chest wall
E) Oral administration of antibiotics for 7 days

A

The correct response is Option A.

This patient’s presentation is consistent with late radiation tissue injury (LRTI), which occurs in 5 to 15% of long-term cancer survivors who have received radiation and can occur months to years (even decades) after treatment. It can vary significantly with age, dose, and site of treatment. Characteristics include progressive tissue deterioration secondary to decreased vascularity followed by replacement of normal soft-tissue architecture by dense fibrotic tissue until there is insufficient oxygen delivery to sustain normal function. When LTRI of the chest wall progresses to skin breakdown and ulceration, as has happened in this patient, a biopsy is always required to rule out recurrence of the primary tumor or a radiation-induced squamous cell carcinoma or soft-tissue sarcoma.

Hyperbaric oxygen therapy has been proposed as a treatment modality that can improve tissue quality and prevent tissue breakdown in irradiated areas. While most data on this relate to treatment of osteoradionecrosis of the mandible, there have been some reports of its use on the chest wall after irradiation due to breast cancer. However, the question specifically asks for the most appropriate next step, which would be biopsy rather than hyperbaric oxygen therapy.

The principles of management once malignancy has been ruled out include debridement of necrotic tissues (including ribs) and reconstruction with well-vascularized flaps. In this case, a left latissimus muscle flap and skin graft was used for reconstruction after aggressive debridement. No alloplastic material was placed or thoracic cage reconstruction performed. This is common in these types of patients as excessive fibrosis caused by the radiation to the chest wall prevents loss of respiratory efficiency through paradoxical motion.

CT scan or MRI may be useful in evaluating the size, extent, and nature of the problem, but does not take priority over tissue biopsy.

The redness that is visualized on the patient’s skin in the case is related to post-radiation changes and telangiectasias, which are common in LTRI, and is not infection. Therefore, antibiotics would be unnecessary.

82
Q

A 34-year-old woman is scheduled to undergo bilateral prophylactic mastectomy because of a strong family history of breast cancer. Reconstruction with free flaps from the upper inner thigh is planned. Which of the following is the pedicle to these flaps?

A) Ascending branch of the lateral femoral circumflex artery
B) Descending branch of the lateral femoral circumflex artery
C) Medial femoral circumflex artery
D) Superficial circumflex iliac artery
E) Superficial femoral artery

A

The correct response is Option C.

The transverse upper gracilis (TUG) flap is a free flap that can be used in breast reconstruction. It is particularly useful in cases where the abdominal donor site is not available or not desired. The flap incorporates the skin and subcutaneous tissues of the upper inner thigh, in the region of the medial thigh lift. The TUG flap is based on the gracilis vessels and perforators from the descending branch of the medial femoral circumflex artery, which forms the dominant pedicle. This arises from the profunda femoris artery. Modifications of this flap have been proposed to increase the volume available for free tissue transfer.

The ascending branch of the lateral femoral circumflex artery provides blood supply to the tensor fascia lata flap.

The descending branch of the lateral femoral circumflex artery forms the pedicle for the anterolateral thigh flap.

The superficial circumflex iliac artery forms the pedicle for the groin flap.

Branches from the superficial femoral artery provide minor pedicles to the distal portion of the gracilis muscle, but are less important and distal to the area of the TUG flap.

83
Q

A 41-year-old woman comes to the office because of an invasive ductal carcinoma of the left breast. On mammography, the tumor is 3 cm from the nipple and measures 4 cm. A left-sided lateral periareolar scar extending from the 12 o’clock to the 3 o’clock position from a previous biopsy is noted. The patient wishes to undergo a nipple-sparing mastectomy. Which of the following findings places the patient at greatest oncologic risk, including risk for de novo or recurrent cancer or inadequate surgical margins, with this procedure?

A) Distance of tumor to nipple
B) Patient age
C) Presence of the periareolar scar
D) Size of tumor
E) Type of tumor

A

The correct response is Option D.

As surgical approaches to breast cancer treatment have evolved, nipple-sparing mastectomy (NSM) has emerged as an alternative to other approaches. It was initially used for prophylactic mastectomies, and patients reported increased satisfaction and body image with nipple-areola complex (NAC) preservation. The role of NSM has been expanded to therapeutic mastectomy, and with that there has been increased research in the oncologic safety of this approach.

Studies have evaluated therapeutic NSM in the context of invasive ductal carcinoma, invasive lobular carcinoma, and ductal carcinoma in situ. The type of cancer does not appear to be associated with the oncologic safety of NSM. Several studies have demonstrated an inverse association between NAC involvement and distance of the tumor from the nipple. While these studies have varied in their distance cutoffs, nipple involvement is reported to be over 50% when the tumor-nipple distance is less than 2 cm, as noted in one study. There is a direct correlation between tumor size and NAC involvement—the same study cited data that when the tumor was greater than 4 cm, the likelihood of nipple involvement was greater than 50%.

One published screening algorithm for plastic surgery includes tumor size less than 3 cm, and tumor location greater than 2 cm from the nipple as criteria for NSM candidacy.

A periareolar scar, if large, may compromise the blood supply to the NAC. Acceptable incisions for NSM, however, include a periareolar incision of 25 to 50%.

84
Q

A 26-year-old woman with a strong family history of breast cancer undergoes genetic testing. She is found to have a deleterious mutation of the BRCA1 gene. Which of the following best describes her lifetime risk for the development of breast cancer when compared with women without this mutation?

A) Two times greater
B) Six times greater
C) Ten times greater
D) Fifteen times greater
E) Twenty times greater

A

The correct response is Option B.

A woman’s lifetime risk of developing breast and/or ovarian cancer is greatly increased if she inherits an altered BRCA1 or BRCA2 gene. Women with an inherited alteration in one of these genes have an increased risk of developing these cancers at a young age (before menopause) and often have multiple close family members with the disease. These women may also have an increased chance of developing colon cancer.

Men with an altered BRCA1 or BRCA2 gene also have an increased risk of breast cancer (primarily if the alteration is in BRCA2) and possibly prostate cancer. Alterations in the BRCA2 gene have also been associated with an increased risk of lymphoma, melanoma, and cancers of the pancreas, gallbladder, bile duct, and stomach in some men and women.

According to estimates of lifetime risk, approximately 13.2% (132 of 1000 individuals) of women in the general population will develop breast cancer, compared with estimates of 36 to 85% (360 to 850 of 1000) of women with an altered BRCA1 or BRCA2 gene. In other words, women with an altered BRCA1 or BRCA2 gene are 3 to 7 times more likely to develop breast cancer than women without alterations in those genes. Lifetime risk estimates of ovarian cancer for women in the general population indicate that 1.7% (17 of 1000) will get ovarian cancer, compared with 16 to 60% (160 to 600 of 1000) of women with altered BRCA1 or BRCA2 genes. No data are available from long-term studies of the general population comparing the cancer risk in women who have a BRCA1 or BRCA2 alteration with women who do not have an alteration in these genes. Therefore, these figures are estimated ranges that may change as more research data are added.

85
Q

A 42-year-old woman comes to the office because she is dissatisfied with an obvious step-off between the chest wall and the superior pole of the breast 6 months after she underwent immediate expander reconstruction of the left breast. No further adjuvant therapy was indicated. After full expansion, the tissue expander was removed, and a permanent smooth, round silicone prosthesis was placed. BMI today is 28 kg/m2. Examination shows a well-defined inframammary fold. The volume is matched to the contralateral breast. Which of the following is the most appropriate surgical procedure for correction of this patient’s deformity?

A) Exchange of the silicone prosthesis with a silicone prosthesis of a larger volume and dimension
B) Exchange of the silicone prosthesis with a saline prosthesis of the same volume and dimension
C) Fat grafting to the superior pole and chest wall
D) Placement of acellular dermal matrix as an inferolateral sling
E) Removal of the prosthesis and reconstruction with a deep inferior epigastric perforator flap

A

The correct response is Option C.

Fat grafting has been shown to successfully address acquired contour deformities in breast reconstruction. Changing the prosthesis to a saline prosthesis of the same size and dimension will not address the problem of the step-off between the prosthesis and the chest wall in this slender patient (BMI 20 kg/m2) which is due to decreased soft-tissue coverage in the superior pole. Likewise, increasing the size and dimensions of the silicone prosthesis would not correct this contour deformity and would also lose the symmetry with the contralateral breast. This patient, without excess lower abdominal skin and subcutaneous fat, is not a candidate for a deep inferior epigastric perforator flap. Placement of acellular dermal matrix as an inferolateral sling is typically performed in the initial stage of tissue expansion and reconstruction. It can help define the inframammary fold, which this patient does not need.

86
Q

A 33-year-old woman comes to the office with a 6-cm rapidly growing tumor of the left breast. She wears a size 36C brassiere. The tumor has a bluish hue and skin veins are dilated. A phyllodes tumor is diagnosed, and surgical excision is planned. Which of the following is the most appropriate surgical procedure to treat this patient?

A) Excision with 1-cm margin
B) Excision with 2-cm margin
C) Excisional biopsy
D) Modified radical mastectomy
E) Radical mastectomy

A

The correct response is Option A.

Phyllodes are large benign tumors that occur primarily in the perimenopausal patient. Previously, they were referred to as cystosarcoma phyllodes, a term coined in 1838 because the tumors are fleshy and have a gross leaf-like intracanalicular growth pattern. However, this is a misnomer because these tumors do not behave like sarcomas and are rarely malignant. The histologic characteristics that separate fibroadenomas from phyllodes tumors are not well defined and have been somewhat controversial. Nevertheless, phyllodes tumors typically are large fibroadenomas that histologically have more stromal cellularity than that seen in the typical fibroadenoma. The classification of benign versus malignant phyllodes tumors is not sharply delineated, and the term borderline lesion may be more appropriate. Borderline lesions have more mitoses per high-power field and moderate nuclear pleomorphism. They have a tendency to recur after local excision but do not demonstrate true malignant behavior. When metastases of a phyllodes tumor have been reported, there have been obvious sarcomatous elements such as liposarcoma or rhabdomyosarcoma in the lesion.

The surgical treatment of phyllodes tumors has recently been redefined. In the past, simple or radical mastectomies were recommended for the treatment of large phyllodes tumors. Currently, most surgeons perform more conservative surgery. Several clinical studies have recommended the excision of tumors with 1-cm clear margins or mastectomy if breast conservation is impossible.

87
Q

A 41-year-old man is referred to the office because of a mass on his left breast that has been growing rapidly for 1 month. Examination of the specimen obtained on core needle biopsy confirms invasive mammary carcinoma. Genetic testing results are positive for BRCA2. In addition to an increased risk of male breast cancer, which of the following other types of cancer is most likely to be associated with this patient?

A) Colon
B) Lung
C) Pancreatic
D) Renal
E) Thyroid

A

The correct response is Option C.

Men with harmful BRCA1 mutations also have an increased risk of breast cancer and, possibly, pancreatic cancer, testicular cancer, and early onset prostate cancer. However, male breast cancer, pancreatic cancer, and prostate cancer appear to be more strongly associated with BRCA2 gene mutations. Colon, lung, renal, and thyroid cancers are not associated with the BRCA1 and BRCA2 gene mutations.

88
Q

A 40-year-old woman is referred for reconstruction following mastectomy for a peripherally located ductal carcinoma in situ. A nipple-sparing mastectomy with immediate, single-stage prosthesis reconstruction with acellular dermal matrix is planned. Which of the following interventions is most appropriate to ensure that no residual cancer exists?

A) Chemotherapy
B) Ductal washing
C) Radiation therapy
D) Retroareolar frozen section
E) Sentinel lymph node evaluation

A

The correct response is Option D.

More plastic surgeons are performing reconstruction for women pursuing prophylactic mastectomy, which is requested quite frequently to avoid cancer recurrence and to achieve optimal aesthetic outcome. Exclusion criteria for nipple-sparing mastectomy include tumors greater than 3 cm, clinical invasion of the nipple-areola complex, tumors within 2 cm of the nipple, evidence of multicentric disease, positive intraoperative retroareolar frozen section, or nodal disease. If carcinoma is found in the retroareolar tissue, the nipple-areola complex must be removed. A patient who would require sentinel lymph node evaluation, radiation therapy, or chemotherapy would not be an appropriate candidate for nipple-sparing mastectomy. Annual mammography is recommended for any patient with a history of breast cancer and is not specific to the issue of nipple-sparing mastectomy. Ductal washing is not relevant for this pathology.

89
Q

An otherwise healthy 52-year-old woman is scheduled to undergo bilateral breast reconstruction with abdominal perforator flaps. She does not smoke cigarettes. Which of the following is the most appropriate antibiotic prophylaxis regimen?

A) Preoperative antibiotics
B) Preoperative chlorhexidine bath
C) Preoperative and intraoperative antibiotics
D) Preoperative, intraoperative, and postoperative antibiotics for 7 days
E) Preoperative, intraoperative, and postoperative antibiotics until all drains are removed

A

The correct response is Option C.

For a healthy patient undergoing a lengthy autologous breast reconstruction, the most appropriate regimen of antibiotic prophylaxis consists of a preoperative dose of intravenous antibiotic and intraoperative antibiotics to ?maintain therapeutic levels during operation and, at most, a few hours after closure? (Mangram, et al). Although preoperative chlorhexidine bathing is supported by some studies and strong theoretical rationale, it is not nearly as effective as antibiotic prophylaxis on its own. A single dose of preoperative antibiotic intravenously is appropriate for short procedures; however, therapeutic levels of antibiotic should be maintained throughout the lengthy surgery described. Continued administration of prophylactic antibiotics for 7 days following surgery or until drains are removed has not been proven to reduce the incidence of surgical site infections. This regimen also promotes drug resistance and adverse patient reactions.

90
Q

A 52-year-old woman with breast cancer undergoes right mastectomy and reconstruction with a free transverse rectus abdominis musculocutaneous (TRAM) flap. The procedure is uneventful. In the recovery room, the patient’s husband says that she has been smoking one pack of cigarettes daily up to the day of surgery. Which of the following postoperative complications is most likely to occur?

A) Flap hematoma
B) Mastectomy skin loss
C) Microvascular thrombosis
D) Partial flap loss
E) Superficial infection

A

The correct response is Option B.

Patients who smoke cigarettes and who undergo breast reconstruction with a free flap have a higher rate of mastectomy skin loss. In addition, they have a high rate of donor site abdominal flap necrosis, umbilical necrosis, and hernias. There is no increase in microsurgical complications, flap-related complications (partial flap loss or fat necrosis), infections, or hematomas.

Current recommendations are for patients to stop smoking at least 4 weeks prior to breast reconstruction. Patients who quit smoking prior to this period have a lower risk of perioperative complications when compared to active smokers.

91
Q

A 53-year-old woman is referred for consultation regarding breast reconstruction following mastectomy. Autologous breast reconstruction options are discussed. Which of the following is an advantage of the pedicled transverse rectus abdominis musculocutaneous (TRAM) flap over the free TRAM flap?

A) Decreased complications in patients with a history of smoking
B) Decreased incidence of complete flap loss
C) Decreased risk of abdominal hernia
D) Decreased risk of fat necrosis
E) Ease of insetting and flap shaping

A

The correct response is Option B.

There is considerable controversy regarding the merits of the pedicled TRAM, free TRAM, free deep inferior epigastric perforator (DIEP), and free superficial inferior epigastric perforator (SIEP) flap reconstructions. Reconstruction with each flap has distinct advantages and disadvantages. Currently, the literature does not clearly favor the use of one procedure over the others.

The pedicled TRAM flap is characterized by a shorter operative time and a decreased risk of complete flap loss when compared to the free flaps. However, the pedicled TRAM flap shows a higher incidence of fat necrosis with partial flap loss, as well as an increased length of stay. Free flaps offer ease of flap shaping and insetting, as well as a decreased risk of abdominal hernia and abdominal wall weakness. Free flaps are more suitable for patients who are diabetic, are overweight, or smoke cigarettes.

Deciding which particular procedure to perform requires assessment on a case-by-case basis and consideration of the surgeon’s level of comfort with the different techniques.

92
Q

A 45-year-old woman with a T2 N0 M0 invasive ductal carcinoma in the inferior pole of the left breast is scheduled to undergo segmental mastectomy followed by radiation therapy. She currently wears a size 36E brassiere and wants to have any cup size from a C to D. After segmental resection of the tumor, which of the following procedures is most likely to yield the best cosmetic result in this patient?

A) Bilateral reduction mammaplasty
B) Primary closure of the left breast and reduction mammaplasty of the right breast
C) Reconstruction of the left breast with a latissimus dorsi musculocutaneous flap and reduction mammaplasty of the right breast
D) Reconstruction of the left breast with a transverse rectus abdominis musculocutaneous flap and mastopexy of the right breast
E) A single-stage breast reconstruction with placement of a silicone prosthesis in the left breast and reduction mammaplasty of the right breast

A

The correct response is Option A.

In a patient with large breasts, in whom a partial mastectomy is required, reduction mammaplasty is an appropriate management. This procedure will potentially relieve symptoms of macromastia, reduce the amount of breast tissue present in both breasts, and offer the best aesthetic outcome. Implantation of a prosthesis in a breast that will undergo radiation therapy increases the risk for complications and would likely lead to a less symmetrical result. The latissimus dorsi or transverse rectus abdominis musculocutaneous (TRAM) flaps could be used for immediate partial reconstruction, but they are ideal for delayed reconstruction of partial mastectomy defects. Reduction mammaplasty does not preclude future reconstruction options, but a latissimus flap reconstruction would. With a T2 tumor, a significant portion of the lower pole of the breast is removed to obtain clear margins. Even though the patient described has moderate-to-large breasts, there is a high likelihood that she will develop a deformity of the left breast and asymmetry with the right breast if no reconstruction is performed.

93
Q

A 46-year-old woman with ductal carcinoma in situ is scheduled to undergo right mastectomy. Immediate reconstruction with a tissue expander and acellular dermal matrix (ADM) is planned. Which of the following is an expected outcome with use of ADM?

A) Decreased formation of seroma
B) Decreased incidence of hematoma
C) Decreased risk of infection
D) Increased capsule contracture
E) Increased initial fill of the expander

A

The correct response is Option E.

According to Sbitany, et al, acellular dermal matrix (ADM) allows for a greater initial fill of saline. This potentially improves cosmetic outcome, as it better capitalizes on preserved mastectomy skin for reconstruction. Sbitany, et al, concluded that ADM-assisted prosthesis breast reconstruction has a safety profile no worse than that of complete submuscular coverage but offers the benefit of fewer expansions and the potential for more predictable secondary revisions.

ADM has enhanced prosthesis-based reconstruction and remains useful in immediate prosthetic breast reconstruction. However, it has been found to have higher rates of postoperative seroma and infection. It has also been reported to decrease capsule contracture.

94
Q

A 42-year-old woman is evaluated because of an invasive cancer of the right breast. Which of the following best approximates the likelihood that this patient’s cancer is associated with the BRCA1 or BRCA2 genes?

A) 1%
B) 10%
C) 25%
D) 40%
E) 75%

A

The correct response is Option B.

Among familial breast cancers, 5 to 10% are considered to be hereditary. These familial breast cancers are linked to specific mutations on a cancer susceptibility gene. The breast cancer susceptibility genes (BRCA) belong to a class of genes known as tumor suppressors. In normal cells, BRCA1 and BRCA2 genes stabilize the DNA and prevent uncontrolled cell growth.

A woman’s lifetime risk of developing breast and/or ovarian cancer is greatly increased if she inherits a mutation on BRCA1 or BRCA2 genes. BRCA1- and BRCA2-related breast cancers occur in younger women and are often associated with estrogen receptor-negative tumors.

95
Q

A 37-year-old woman comes to the office for consultation regarding left breast reconstruction after mastectomy to treat breast cancer. Chemotherapy and radiation therapy are planned postoperatively. She wears a size 34D brassiere. Height is 5 ft 6 in (168 cm), and weight is 160 lb (73 kg). BMI is 25.8 kg/m2. She does not want abdominal scars. Autologous reconstruction with a transverse musculocutaneous gracilis (TMG) flap is planned. Which of the following is a disadvantage of using a TMG flap for this patient’s reconstruction?

A) Difficulty of flap harvest
B) High risk of donor site morbidity
C) High risk of fat necrosis
D) Small flap size
E) Variable vascular anatomy

A

The correct response is Option D.

For breast reconstruction, the TMG flap offers a valuable alternative therapy compared with other standard flaps from the lower abdomen, such as the transverse rectus abdominis musculocutaneous (TRAM) or DIEP flaps. The amount of tissue that can be transferred, however, is limited. The largest flap harvested in one large series weighed 420 g and measured 30 × 10 cm. The flap usually offers enough volume to reconstruct small- to mid-sized breasts. The major advantage of this flap as compared with other flaps, such as the gluteal flap or the perforator latissimus flap, is its constant vascular anatomy. Flap perfusion is always reliable, with low rates of fat necrosis and tissue similar in consistency with breast tissue. The donor scar is inconspicuous. The incision is comparable to incisions for a thigh lift and is well hidden. A distortion of the labia majora is typically not observed. However, as in thigh lifts, a lowering of the scar remains a possible problem. Functional donor-site morbidity after TMG flap harvest itself is low. After having clinically established the transverse approach for routine procedures, harvesting one flap is rapid and usually takes no longer than 30 minutes.

96
Q

A 43-year-old Caucasian woman is referred to the office because of a mass on her right breast that has been rapidly growing for 8 weeks. Physical examination shows a 4.5-cm, freely movable mass in the right breast. No axillary adenopathy or nipple discharge is noted. Which of the following is the most likely nonepithelial neoplasm in this patient?

A) Fibroadenoma
B) Hamartoma
C) Lipoma
D) Phyllodes tumor
E) Primary breast lymphoma

A

The correct response is Option D.

Primary breast lymphoma is rare; it constitutes less than 0.6% of all breast malignancies. Phyllodes tumors represent about 1% of tumors in the breast; they are the most commonly occurring nonepithelial neoplasm of the breast. The tumor has a smooth, sharply demarcated texture and is generally freely movable. It is a relatively large tumor, with an average size of 5 cm. A hamartoma is the most common benign tumor of the lung. Hamartomas of the breast are benign tumors composed primarily of dense fibrous tissues with variable amounts of fat and associated ducts. Eighty-five percent of phyllodes tumors are benign, and 15% are malignant. There is no race predilection, but phyllodes tumors occur almost exclusively in women. In young women under age 25 years, asymmetric, tender, and fibrocystic tissues usually point to a fibroadenoma or circumscribed fibrocystic mass. Lipoma of the breast causes diagnostic and therapeutic uncertainty. Clinically, it may be difficult to distinguish a lipoma from other conditions. Fine-needle aspiration cytology is often not helpful. Both mammography and ultrasonography results are often negative. MRI may be overread, but it is useful as a diagnostic tool. This neoplasm can be seen in men and women.

97
Q

A 12-year-old girl with Beckwith-Wiedemann syndrome develops profound breast enlargement at the onset of puberty. Physical examination shows two distinct masses in the right breast and severe hyperplasia consuming the left breast. A photograph is shown. Which of the following is the most appropriate surgical management?

A) Hormone suppression therapy
B) Right lumpectomies; left mastectomy with skin reduction with application of nipple as skin graft
C) Right simple mastectomy; left mastectomy with skin reduction with application of nipple as skin graft
D) Right simple mastectomy; left simple mastectomy with sentinel lymph node biopsy
E) Right total mastectomy; left modified radical mastectomy

A

The correct response is Option B.

Epithelial hyperplasia is a benign pathological process. Therefore, modified radical mastectomy or sentinel lymph node biopsy would not be indicated in the absence of cancer. Mastectomy on the right is not indicated because the masses are discrete and separate from the normal breast parenchyma. As such, a viable breast mound can be preserved in this 12-year-old girl. Because of the expansive nature of her hyperplasia on the left, skin reduction is required to ensure smooth contour of her skin flaps. The nipple may be spared because of the benign disease. Reconstruction can be performed in a delayed fashion after development is complete. An adjustable expander/prosthesis can also provide a reasonable breast mound until definitive reconstruction is performed.

Hormone suppression therapy may temporarily slow the growth of the hyperplasia but could adversely affect this child’s normal development. It would not comprise definitive management.

98
Q

A 60-year-old woman receives low-molecular-weight heparin (LMWH) 40 U subcutaneously 1 hour before undergoing breast reconstruction using a unilateral transverse rectus abdominis musculocutaneous (TRAM) flap. Weight is 185 lb (84 kg); BMI is 32 kg/m2. Which of the following is the most likely effect of the LMWH on perioperative risks in this patient?

A ) Decreased risk of flap failure

B ) Decreased risk of postoperative hematoma

C ) Decreased risk of pulmonary thromboembolism

D ) Increased risk of blood transfusion

E ) Increased risk of infection

A

The correct response is Option C.

There are no uniform standards or guidelines for the routine use of chemoprophylaxis of venous thromboembolism in plastic surgery. At a minimum, lower extremity mechanical compression devices should be used on all patients undergoing general anesthesia. The patient described has multiple risk factors for developing deep venous thrombosis (DVT) and pulmonary embolism, such as an age over 50, prolonged surgery time, malignancy, and being overweight.

In such high-risk patients, data support the use of chemoprophylaxis with either standard heparin or low-molecular-weight heparin given subcutaneously. As the risk of developing DVT begins with anesthesia induction, it is generally recommended that heparin therapy be started before surgery. Out of concern for bleeding, some surgeons start heparin therapy in the early postoperative period. While more data are needed to clarify the optimal start time of therapy, it is clear that starting therapy before surgical incision is generally safe when dosed appropriately.

In large retrospective studies, chemoprophylaxis patients were not more likely to require blood transfusion, though they do demonstrate a slightly greater decrease in postoperative hemoglobin as compared with control patients. There is clearly a decreased risk of postoperative DVT and pulmonary embolism, both clinically apparent and asymptomatic. The data regarding postoperative hematoma are less clear, with some studies showing no increase in €œtake-backs € to the operating room for hematoma with chemoprophylaxis. One study did show an increase in hematomas in oncologic breast surgery with LMWH versus standard, unfractionated heparin. There is no association between flap survival or infection and subcutaneous heparin use.

Until reliable prospective, randomized data of sufficient study size are available to demonstrate optimal treatment, the studies seem to support the routine use of mechanical and perioperative subcutaneous heparin prophylaxis in high-risk patients.

99
Q

A 53-year-old woman comes to the office because of ulcerated tissue 6 weeks after undergoing radiation therapy for breast cancer. She underwent mastectomy 1 year ago. Analysis of the radiated tissue is most likely to show an increase in which of the following?

A ) Acute inflammatory response

B ) Cytokines and growth factors

C ) Neutrophil function

D ) Tissue oxygenation

E ) Vessel thrombosis

A

The correct response is Option E.

Analysis of radiated tissue will show increased vessel thrombosis. Ionizing radiation directly damages the genome or injures the DNA through free radical production. In acute radiation injury, the skin becomes erythematous and edematous with the dilation of fine blood vessels, endothelial edema, and lymphatic obliteration. Although perfusion of the skin assessed with fluorescein injection appears normal, tissue oxygenation is inadequate. Healing is impaired with slowed fibroblast proliferation and impairment of the acute inflammatory response. Fibroblast defects are the main problem in the inhibited healing of chronic radiation injury. Phagocytosis and bacteriocidal metabolic functions in neutrophils are also impaired. This effect increases after therapy, which cannot be a direct effect of radiation on the neutrophils because their lifespan is too short. The local wound environment and irradiated tissue do not prime the neutrophils with the appropriate cytokines and growth factors needed for activation. This may lead to an increased incidence of postoperative infections in patients with previous radiation.

Recent studies have shown promise in the treatment of radiation tissue damage with lipoaspirate transplantation. Adipose-derived stem cells have been hypothesized to target damaged areas, release angiogenic factors, form new vessels, and increase tissue oxygenation.

100
Q

A 45-year-old woman who has breast cancer comes to the office for consultation regarding bilateral breast reconstruction. Reconstruction using autologous abdominal tissue is considered. The risk of abdominal morbidity is discussed. Which of the following flap techniques is most likely to result in the lowest level of overall abdominal morbidity?

A ) Deep inferior epigastric artery perforator

B ) Free muscle-sparing transverse rectus abdominis musculocutaneous (TRAM)

C ) Free TRAM

D ) Pedicled TRAM

E ) Superficial inferior epigastric artery flap

A

The correct response is Option E.

The superficial inferior epigastric artery (SIEA) flap results in the lowest level of overall abdominal morbidity, as the technique used in harvesting this flap leaves the abdominal fascia intact. These vessels are only present in less than one third of patients, and only one half of those patients will have vessels of sufficient diameter to support a free tissue transfer. The SIEA flap is also associated with a higher frequency of total flap loss, in addition to a higher incidence of fatty necrosis.

The deep inferior epigastric artery perforator (DIEAP or DIEP) flap involves dissection of one or two (occasionally more) perforators through the rectus muscle to the inferior epigastric vessels. Although this technique does not include any rectus muscle or sheet/fasica within the flap itself, it does involve moderate-level trauma to those organs and can cause abdominal wall morbidity.

The free TRAM or free muscle-sparing TRAM techniques are free tissue transfer variants of the TRAM, whereby a small amount of the rectus is taken with the flap. In the more advanced MS free TRAM technique, the amount of muscle taken is only enough to allow safe transfer of the perforators. Although this technique is less invasive than the pedicled TRAM, it does still involve removal of a variable portion of the rectus muscle and fascia.

A pedicled TRAM flap technique involves transferring of the flap based on the superficial epigastric vessel that runs within the rectus muscle. Therefore, the entirety of the rectus (unilateral or bilateral) is elevated out of its native abdominal wall location.

Many publications have compared the other modes of breast reconstruction with reference to abdominal wall morbidity. This area remains controversial. The general consensus remains that

in a bilateral reconstruction, pedicled TRAM flaps are associated with higher levels of overall abdominal morbidity (hernias, bulges, weakness, intolerance to exercise, etc) when compared with the use of MS free TRAM, DIEP, or SIEA flaps. The use of SIEA flaps results in minimal to no abdominal wall morbidity.

101
Q

A 45-year-old man comes to the office for consultation regarding breast cancer after undergoing gene testing and learning that he is a carrier of the BRCA2 gene mutation. He has a strong family history of breast, prostate, and ovarian cancers. Which of the following best represents his lifetime risk for developing breast cancer?

A ) 6%

B ) 15%

C ) 30%

D ) 60%

E ) 85%

A

The correct response is Option A.

Since the identification of the first breast cancer gene in 1990, the field of molecular breast cancer testing has grown enormously. It is estimated that approximately 5% to 10% of patients diagnosed with breast cancer last year carry highly penetrant cancer susceptibility genes. It is important to identify these patients early, as the presence of genetic mutations or other high-risk factors may alter management strategies for patients, both prior and subsequent to the diagnosis of breast cancer.

Male breast cancer accounts for less than 1% of all breast cancer cases in the United States and is more common in men with a family history of breast cancer. The relationship between male breast cancer and a deleterious BRCA2 mutation has been well established. In the largest study to date, the lifetime risk of developing breast cancer in a BRCA2 male carrier is approximately 7%. The correlation with BRCA1 and male breast cancer is less clear, but it seems that the presence of BRCA1 mutation confers a lower lifetime risk of developing breast cancer than a BRCA2 mutation. The relative risk of developing breast cancer is highest for men in their 30s and 40s, and it decreases with increasing age.

The lifetime risk for the development of breast cancer in female carriers of BRCA1 and BRCA2 mutations is significantly higher than for males. Female BRCA1 carriers have an 85% risk of developing breast cancer and a 62% risk of developing ovarian cancer. Female BRCA2 carriers have an 85% risk of developing breast cancer and a 25% risk of developing ovarian cancer.

102
Q

A 21-year-old woman comes to the office for consultation regarding a palpable lump in the left breast that she first noticed 4 months ago. Phyllodes tumor of the breast is diagnosed. Which of the following is the most appropriate management at this time?

A ) Observation and incisional biopsy

B ) Neoadjuvant chemotherapy

C ) Local excision and annual surveillance

D ) Wide local excision and radiation therapy

E ) Mastectomy with immediate reconstruction

A

The correct response is Option C.

Phyllodes tumors of the breast were once called cystosarcoma phyllodes because of the fleshy nature of the tumor. Because most tumors are benign, the name may be misleading. Thus, the favored terminology is now phyllodes tumor.

Phyllodes tumor is the most commonly occurring nonepithelial neoplasm of the breast but represents only about 1% of tumors in the breast. It has a sharply demarcated, smooth texture and is typically freely movable. It is a relatively large tumor; the average size is 5 cm. However, lesions greater than 30 cm have been reported. Approximately 90% of phyllodes tumors are benign and approximately 10% are malignant.

In considering excision, the tumor-to-breast ratio should be small enough to allow for segmental excision of the tumor and possible reduction mammaplasty technique (eg, inferior pedicle for a superior tumor or a superior pedicle for an inferior lesion). Annual surveillance and follow-up is recommended because the pathologic appearance does not always predict the clinical behavior.